Tuesday, October 29, 2013

Can machines beg the question?


I thank Robert Oerter for his further reply to my recent comments (here, here, and here) on his critique of James Ross’s argument for the immateriality of the intellect.  You will recall that, greatly oversimplified, Ross’s argument is: (A) All formal thinking is determinate, but (B) No physical process is determinate, so (C) No formal thinking is a physical process.  You will also recall that Ross makes use of thought experiments like Kripke’s “quus” example to argue that given only the physical properties of a system, there can be no fact of the matter about whether the system is applying modus ponens, squaring, adding, or computing any other function.  That is what he means by saying that “no physical process is determinate.”  Finally, you’ll recall that among Oerter’s criticisms is that he thinks Ross is being inconsistent.  If we consider Hilda, a human being who can add -- or, as Oerter puts it in his latest post, who can ETPFOA (“execute the ‘pure function’ of addition”) -- then Ross’s argument would, Oerter says, apply to Hilda just as much as to a machine.  Yet Ross, Oerter claims, applies it to the machine but not to Hilda.  Hence the alleged inconsistency.

Oerter’s latest post summarizes his point as follows:

The logic of my Hilda example is straightforward. Ross says that humans can ETPFOA. Ross says that  A, B, and C entail that a computer cannot ETPFOA. I claim that A, B, and C are true for Hilda, too. So A, B, and C entail that Hilda cannot ETPFOA.

With this contradiction, the whole argument falls to pieces. Now, you can argue that I am wrong: that A, B, and C are not true of Hilda. Or you can argue that there is some D that I missed that is true of the computer but not true of Hilda. But you can't say this example is irrelevant to the soundness of Ross's argument.

End quote.  The problem, of course, is that Oerter is blatantly begging the question here.  A, B, and C entail that a computer cannot ETPFOA given the further premise that a machine is purely physical.  And that is a premise that both sides agree on.  But A, B, and C would entail that Hilda cannot ETPFOA only given the further premise that Hilda is purely physical.  And that is something both sides do not agree on; indeed, it is the whole point at issue.

The irony is that Oerter accuses Ross (or at least a reader who defends Ross) of begging the question.  But Ross is doing no such thing.  He would be begging the question in a way parallel to Oerter’s blatant begging of the question only if the further premise he needed was the premise that Hilda is not purely physical.  But that is not the premise he appeals to, and it is not the premise he needs.  Rather, what he needs and what he appeals to is the further premise that Hilda engages in formal thinkingThat premise together with A, B, and C is what generates the conclusion -- not a question-begging assumption but rather a demonstrated result -- that Hilda is not purely physical.

A, B, and C are, after all, only the heart of Ross’s position.  A little more fully spelled out, his overall argument essentially goes something like this:

A. All formal thinking is determinate.

B. No physical process is determinate.

C. No formal thinking is a physical process. [From A and B]

D. Machines are purely physical.

E. Machines do not engage in formal thinking. [From C and D]

F. We engage in formal thinking.

G. We are not purely physical. [From C and F]

The argument is valid, so to undermine it Oerter will have to reject at least one of the premises.  Premise A is one that Oerter has so far not challenged, and Ross defends it by arguing that we cannot coherently deny it.  Premise B is one that Oerter has also so far not done much to challenge.  His strategy was, at first, to suggest (wrongly, as we have seen) that the premise was really epistemological rather than metaphysical.  That failed, and Oerter shifted his focus to trying to argue that Ross was inconsistent in not drawing from B the same conclusion about human beings that he drew about machines.  As we have also seen, that would be irrelevant to the question of whether B is true even if Ross was being inconsistent.  But another thing we have seen is that Ross is not being inconsistent.

So, Oerter has given us no reason to doubt B, and thus he has given us no reason to doubt that Ross has established C.  D, as I have noted, is a premise both sides agree on.  Hence Oerter has also given us no reason to doubt that Ross has established E.  F is a premise which is not only agreed to by both sides -- at least, I assume that Oerter will agree that we engage in formal thinking -- but it is another premise we cannot coherently deny.  Since G follows from these premises -- premises which, again, Oerter has so far given us no reason to doubt -- he has therefore given us no reason to doubt G.  Ross, meanwhile, has given us very good reason -- I would say conclusive reason (for reasons I explain at length in my ACPQ article on Ross) -- to affirm his premises.  Hence he has given us very good reason to affirm G.

So, the score so far is still Ross: 1, Oerter: 0.  1s and 0s being fitting, I guess, given that it’s computers we’re talking about. 

331 comments:

  1. What about a person who knows A and that A implies B but just before concluding B drops dead of an aneurysm or gets hit by a truck?

    Evidently this person has failed to consistently apply modes ponens. Is the argument actually that silly? That formal thinking would be impossible in a world in which nothing was certain?

    Surely, regardless of whether the world has immaterial aspects or it doesn't, certainty is thin on the ground.

    It is possible that I am the only person in the world who has ever gotten the wrong answer on a math problem, but anecdotal evidence suggests that people not only occasionally fail to reason correctly but that they do so far more frequently than quantum mechanics or thermodynamics would suggest. More frequently in fact that computers would if formal reasoning was a thing that they did.

    So it seems to me a bit of a stretch to assert that formal thinking is deterministic and that human beings think formally. It is not that these two observations are false so much as they miss a step. In order for the logic to actually pass muster the second postulate would have to be something like "human beings deterministically reason formally". Which is to say that humans could reason formally and they could still be indeterminate if they only sometimes reasoned formally and at other random times failed to.

    ReplyDelete
  2. @reighley:

    "So it seems to me a bit of a stretch to assert that formal thinking is deterministic[.]"

    I think you mean determinate here. And of course formal thinking is determinate; even if we make a mistake in reasoning, it's a mistake in reasoning, because reasoning is what we're doing.

    ReplyDelete
  3. Actually, the rest of reighley's post suggests that he did take Prof Feser to mean deterministic.

    That is, of course, not the argument, for reasons enumerated in the first post of the recent exchange:

    edwardfeser.blogspot.com/2013/10/oerter-and-indeterminacy-of-physical.html

    ReplyDelete
  4. @Greg:

    Well, then, he didn't mean determinate, but he should have.

    ReplyDelete
  5. Can some kind person explain, in terms suited to a motivated non-initiate, what "determinate" means as it is used here?

    ReplyDelete
  6. Right, he seems to be making the same mistake Oerter made.

    ReplyDelete
  7. Well, that's five comments wasted.

    reighley, "determinate" in the present context has absolutely nothing to do with "determinism" in the causal sense. We went through that four posts ago in this series.

    ReplyDelete
  8. Right you are! I am confused about the terms again. To be honest every time this subject comes up it seems like a relationship between causation and logical implication is being hinted at but never quite stated.

    So I went back and found the term I was supposed to mean :
    "There’s nothing in the physical properties of Δ that entails any of these interpretations, or any other for that matter. The physical properties are “indeterminate” in the sense that they don’t fix one particular meaning rather than another."

    and from the first page of the ACPQ article :
    "Adding single instances even to infinity cannot exclude exclude incompossible equally most particular forms (cf Saul Kripke's plus/quus example)."

    I wish I could salvage the examples about the intervention of chance in reasoning but obviously I can't.

    Isn't the conclusion still true though? If I am presented with person who most of the time thinks 1+2=3 and then sometimes, quite by mistake thinks 1+2=4 am I really justified in asserting that this train of thought has "a definite abstract form".

    ReplyDelete
  9. @reighley:

    As I said in one of the wasted posts, a mistake in reasoning is still a mistake in reasoning, because reasoning is what we're trying to do.

    Failure to add perfectly doesn't mean our intellect isn't instantiating addition any more than a defect in a squirrel (having only three legs, say) means that it doesn't instantiate the form of a squirrel.

    ReplyDelete
  10. @Scott,
    then why is it that we are willing to allow a physical system to specify a squirrel, but not to specify modes ponens?

    Something which is close enough to being a squirrel is a squirrel in fact. Addition which fails only on some impossibly large numbers is not quaddition but simply addition in error.

    Or is the argument rather that physical variables alone cannot determine whether or not something is a squirrel?

    ReplyDelete
  11. @reighley:

    The argument is that physical variables alone aren't sufficient to determine that a physical system is adding. In the case of human beings exercising our intellects, we know that we are adding, so we also know that there's something non-physical involved. This is unaffected by the fact that we sometimes add badly; we're still adding badly rather than doing something else.

    Likewise, the adding machine is also really adding, though only in a derivative or analogical sense, because that's the task its human designers created it to do and because that's what its users are using it for. This sort of addition, too, is determined by something other than the machine's purely physical states and behavior.

    But it makes no difference to the argument if it turns out either (a) that the machine isn't adding at all, or (b) that the "machine" actually possesses an intellect and is adding in a non-derivative sense. All that matters to the argument is that this isn't determined by the physical facts alone, as those are always consistent with more than one pure function or operation.

    For more, please reread the posts that have preceded this one in the series.

    ReplyDelete
  12. (And don't get carried away with the squirrel analogy. All I meant was that something can instantiate a form imperfectly.)

    ReplyDelete
  13. A. All formal thinking is determinate.

    I don't think it is. Modus ponens is an example of formal thinking, but we can exchange and re-interpret the terms and still have a valid argument. If you allow symbols to be re-interpreted or left indeterminate, the structure of the formal argument does not determine them. It constrains the interpretations, but does not specify them uniquely.

    Arithmetic works similarly. 2+3=5 means that two apples and three apples is the same as fives apples, and it means two oranges and three oranges are the same as five oranges. It doesn't determine what sort of objects are being operated on.

    Abstraction is precisely the process of presenting a formal argument in an indeterminate way, because that gives us the power to simultaneously prove a large number of different statements, with different meanings, with a single step.

    B. No physical process is determinate.

    Physical processes can be as determinate as formal thinking. Modus ponens and similar steps can be performed automatically and by rote. It was in hopes of being able to do so that they were first invented.

    And machines can as easily insert concrete propositions into a formal argument template as a human can.

    Such constraints on meaning as formal thinking is *able* to impose, the automatic version imposes too.

    ReplyDelete
  14. @NiV:

    "Modus ponens is an example of formal thinking, but we can exchange and re-interpret the terms and still have a valid argument. If you allow symbols to be re-interpreted or left indeterminate, the structure of the formal argument does not determine them. It constrains the interpretations, but does not specify them uniquely.

    Arithmetic works similarly. 2+3=5 means that two apples and three apples is the same as fives apples, and it means two oranges and three oranges are the same as five oranges. It doesn't determine what sort of objects are being operated on."

    Neither of those is an example of formal indeterminacy. All modus ponens arguments share the same (determinate) form no matter what their terms, and addition is (determinately) addition quite independently of the natures of the objects being added. (Strictly speaking we don't add "objects" anyway; we add numbers. And two is two.)

    ReplyDelete
  15. @NiV:

    "Physical processes can be as determinate as formal thinking. Modus ponens and similar steps can be performed automatically and by rote."

    You're missing the point here too, I'm afraid. Yes, in one (derivative) sense the movements of (say) an abacus or the changes of voltage in a computer can be regarded as determinate in the relevant sense: the abacus or computer is really adding, because we're really adding when we design and use the device for that purpose. But that these movements/changes instantiate the formal operation of addition is not just a matter of the physical details, because those physical details are always consistent with quite a lot of incompossible operations.

    ReplyDelete
  16. "and addition is (determinately) addition quite independently of the natures of the objects being added."

    Unless you're adding logarithms! :-)

    ReplyDelete
  17. Even then you're still adding numbers and the formal operation is the same; you just happen to be doing it in order to perform a multiplication.

    And if you're conceding that the ultimate operation is "multiplication" because I'm regarding the two numbers I'm adding as logarithms in some suitable base—well, then, I'm happy to hear it. Otherwise we'd be multiplying every time we added, and I don't think you're about to claim that.

    ReplyDelete
  18. "Otherwise we'd be multiplying every time we added, and I don't think you're about to claim that."

    Oh, but I am!

    Both calculations are performed simultaneously, by a single mechanical process. Adding numbers always simultaneously multiplies their exponentials. Both meanings are implicit in the action.

    And I quite often *intend* the action to be both.

    ReplyDelete
  19. "And I quite often *intend* the action to be both."

    Well, it isn't just "both"; the two (positive real) numbers you're adding can be understood as exponents in any (positive real) base, so on your view you're actually multiplying an uncountable infinity of number-pairs every time you add. Moreover (also on your view), it's irrelevant whether you intend to do so.

    ReplyDelete
  20. Scott,

    Yes. That's right.

    ReplyDelete
  21. Is there a critique out there of Ross's argument that doesn't wildly misunderstand it? Every opposing voice in the comboxes and forums has been so far off the mark as to be of no edifying value whatsoever. I'd like to hear how materialists who understand the argument resist its conclusion.

    ReplyDelete
  22. @Chad Handley:

    "Is there a critique out there of Ross's argument that doesn't wildly misunderstand it?"

    I'm not aware of any, no.

    But then again, nearly the only thing I ever do on the Internet these days is search frantically to find out when the next issue of Theodicy will be available. The cover looks great.

    ReplyDelete
  23. Amen, Chad! But I think the total failure of so many obviously intelligent people to grasp what is really a pretty simple argument is revealing. I suspect that at least on a subconscious level they fear (with good reason) that to understand the argument is to see that one is rationally compelled to either give up materialism or embrace the absurdity of eliminativism. So many of these comments on the last several posts have amounted to little more than attempts to muddy the waters.

    ReplyDelete
  24. @NiV:

    "Yes. That's right."

    And yet you also deny that formal thinking is determinate.

    ReplyDelete
  25. Haha, thanks, Scott.

    Unfortunately self-publishing a comic book is expensive so I can't keep a monthly pace right now.

    Also, I just found out the local comic book stores have been telling people that the book is a work of atheist apologetics, which I hear has been scaring people off and further hurting my sales. I guess the cover and first page could lead one to that impression.

    I've been considering a kickstarter; that's the only way I'll get issue 2 out before Christmas, unfortunately.

    ReplyDelete
  26. @Scott,
    "The argument is that physical variables alone aren't sufficient to determine that a physical system is adding. In the case of human beings exercising our intellects, we know that we are adding, so we also know that there's something non-physical involved. This is unaffected by the fact that we sometimes add badly; we're still adding badly rather than doing something else."

    This is the part I don't get. If I am permitted to define "adding" in terms which admit of a substantial number of mistakes, then the space of operations which constitute "adding" is not as particular or fixed as Ross seems to need it to be.

    The thought is no more determinate than the act is. If you say "I am adding", I say "but which adding are you doing? Which mistake are you going to make?".

    Your thoughts determine nothing precisely. Nor do I see why they have to. Determining things to a sufficiently fine approximation seems to be good enough for everything else that passes as knowledge.

    So it is the first premise that lost me, that formal thinking is determinate.

    ReplyDelete
  27. If I am permitted to define "adding" in terms which admit of a substantial number of mistakes, then the space of operations which constitute "adding" is not as particular or fixed as Ross seems to need it to be.

    But you wouldn't be defining 'adding' in terms which admit of a substantial number of mistakes: in order for them to be mistakes at all, the adding intended has to be adding in the strict mathematical sense. They are only mistakes relative to what addition strictly is, which has to be what you were intending, or you wouldn't have actually made a mistake.

    ReplyDelete
  28. @Chad

    Ooooohhhhh, you've just made me dislike local comic book stores. Lol. All the garbage out there and they run down your comic? I'll keep you and your work in my prayers. Hopefully I'll see you on TV someday and be able to say "I prayed for that guy when he was just starting out."

    Eric

    ReplyDelete
  29. "And yet you also deny that formal thinking is determinate."

    Yes. Your point?

    Anonymous,

    "Amen, Chad! But I think the total failure of so many obviously intelligent people to grasp what is really a pretty simple argument is revealing"

    :-)

    And I was puzzled as to why so many non-materialists had such difficulty grasping the pretty simple counter-argument! Perhaps they're afraid of the conclusion?

    reighley,

    "This is the part I don't get. If I am permitted to define "adding" in terms which admit of a substantial number of mistakes, then the space of operations which constitute "adding" is not as particular or fixed as Ross seems to need it to be."

    The details of the implementation are irrelevant for their argument. No finite machine can add in general, they run out of memory. It wouldn't matter if the operation was to 'scramble the digits' - there are lots of functions that will do that. The bit they're pointing at is that there is something interpreting the operation as addition. Because the only explanation they can imagine for 'interpreting a meaning' requires a ghost in the machine, the existence of a ghost is thus proved.

    The stuff about it being "determinate" is the attempt to identify what it is the "meaner" does, that an algorithm cannot. I don't think it succeeds, but we're still trying to sort that out.

    ReplyDelete
  30. NiV,

    Is this the counterargument about similarities? Where you claim that when we think of Supa Nova or Cats this is because of a similarity of our neurons, our brain matter, to Supa Novas or to Cats?

    I'm trully astounded that non-materialists are not instantly won over by this counterargument.

    ReplyDelete
  31. The stuff about it being "determinate" is the attempt to identify what it is the "meaner" does, that an algorithm cannot. I don't think it succeeds, but we're still trying to sort that out.

    That's not it, and maybe it's the cause of the confusion. An algorithm is an instance of "formal thinking" and the argument is that no formal thinking is a physical process. The disjunction is between formal thinking and physical process not between meaning and algorithm , the latter of which is merely an instance of the former.

    Algorithms, as an instance of meaning, are abstract since meaning is abstract. This is why the very same algorithm can be coded in computer code, writen on paper, or verbally communicated.

    Now the point of the argument is that formal thinking - in this case the algorithm of addition - cannot be reduced to the purely physical because the purely physical itself cannot specify which algorithm it instantiates among many possible alternatives. But the point of the argument is that it doesn't instantiate any algorithm at all absent a mind to define the algorithm. An abacus whose beads are flipped back and forth by the wind is not adding, even if the sequence of events happens to be the same that a human user would have done had he been using the abacus to add.

    ReplyDelete
  32. Interestingly, it's premise D that I disagree with! If something like panpsychism is true, then no physical system is purely physical, even a computer. All real causation is conscious to some extent.

    So, if the machine had the right causal structure it could indeed have determinate thought. As a possible sci-fi scenario example: a quantum computer may be able to generate enough "physical" indeterminacy to allow for thought and determinate thinking. Thought determines "physical" outcomes through selection or, per QM parlance, "measurement."

    I think this kind of answer is the proper dialectical synthesis from the binary between Feser and Oerter's points of view. (Though if I'm wrong than Feser's right, of course.)

    ReplyDelete
  33. @NiV:

    "Yes. Your point?"

    That if an addition operation actually and literally means all operations of multiplication in which the addends represent exponents in any base, and everything else that could be taken to be homomorphic, that looks pretty danged determinate. It would be indeterminate if it didn't determine any of them as meanings, but determining all of them as meanings is a different matter.

    @Chad Handley:

    I'm having similar issues with a couple of music projects and we're considering Kickstarter as well. I think it's a great idea and I hope it works out for you. Best wishes; I'll be ready to buy it the minute it's available.

    @Matthew Sigl:

    "If something like panpsychism is true, then no physical system is purely physical, even a computer."

    Technically premise D could be read in a way that doesn't contradict this. Under panpsychism, with a strict definition of "machine," the argument would be unaffected, but we'd also conclude (independently) that there are no "machines" in the strict sense.

    However, even under panpsychism it might still be impossible for a machine to have the right "causal structure": even if each of its individual bits possessed a low-grade sort of "inner life," there still might not be a single unified consciousness in the combination and the machine itself still might not be able to engage in determinate thinking. (This is a pretty infamous problem for at least certain versions of panpsychism.)

    ReplyDelete
  34. @Scott - Good points. Totally agree.

    As for what kind of "causal structure" counts, here I agree with the Integrated Information Theory of consciousness by Giulio Tononi who works out a mathematical/geometric model of how a particular causal structure generates a particular state of consciousness. I find it convincing. Ultimately, your consciousness IS a causal structure. You are the selection mechanism - that which determines.

    Check it out:
    http://www.biolbull.org/content/215/3/216.abstract

    ReplyDelete
  35. @reighley:

    Brandon has given the reply I would have given, but I don't want you to think I'm deliberately ignoring your post. As he says, the point is that adding (in the strict mathematical sense) is what I'm trying to do, and it's only relative to that standard that I can be said to fail.

    We needn't regard the failures as "addition," but we do need to regard them as attempts at addition in order to regard them as failures. In order positively to miss a target, I have to have been aiming at it; otherwise I might have been aiming at something else or not aiming at all.

    ReplyDelete
  36. @Matt Sigl:

    Thanks, that looks interesting. I'll give it a read.

    ReplyDelete
  37. @David T:

    Excellent summary, and I do suspect you've identified the root of the confusion.

    @NiV:

    "Because the only explanation they can imagine for 'interpreting a meaning' requires a ghost in the machine, the existence of a ghost is thus proved."

    This isn't correct either, but this subject is spread over several threads and perhaps you haven't read them all.

    As one or two of us have said (I believe Mr. Green was another), the point is that if meaning requires interpretation, then (we argue) the interpretive process must come to an end somewhere. A-T says this happens in an intellect for which simply containing, or being "informed" by, a "form" constitutes understanding, meaning, or what have you, with no further interpretative steps required.

    As for your proposed alternative, I'm still puzzled about precisely what it is and why you think it remotely captures what we mean by "meaning." On your view, if I understand it correctly thus far, everything seems to "mean" pretty much everything else, because everything can be mapped onto pretty much anything else by a suitable choice of homomorphism. And even you want to impose tight restrictions on the allowable sort of homomorphism, it still seems odd to say that two things "mean" one another merely through resembling one another, or that when I "mean" X, I also "mean" everything else holomorphic to X whether I (so to speak) mean to or not.

    ReplyDelete
  38. This is off topic, but I didn't know where else to find quality Thomists.

    I'm wondering if there's any Thomists who would be willing to participate in a formal and respectful debate on debate.org regarding Aquinas' first way. We can decide on the form and everything later. Burden of proof will be on Pro to show that the first way is sound.

    Ben Yachov? Rank sophist? Scott? Anyone else?

    I think it would be fun... I'm busy with life myself, so if you anyone does accept, this isn't meant to be taking the place of that. Just looking for a fun, respectful debate.

    ReplyDelete
  39. @David T,
    "An abacus whose beads are flipped back and forth by the wind is not adding, even if the sequence of events happens to be the same that a human user would have done had he been using the abacus to add."

    The goal is to fix a conception of "adding" or more generally of "formal thinking" in such a way that it is a thing that cannot be said of a physical system but can be said of a human being.

    There are several flavors of this argument (and I am beginning to suspect that I am not the only one who has been confused about which one is on the table when).

    Ross seems to be targeting the relationship between the thought of a person and the thing itself in terms of the cardinality of the subject. Which is to say that when a person thinks of adding they "fix one particular meaning" with emphasis on the one and when a machine tries to add it narrows the field only to a set of "incompossible equally most particular forms".

    If that is the argument then there seem to me to a couple of things wrong with it. The thing that I am bothered about just now is the idea that we have, when thinking about adding, really done the trick of fixing one and only one thing.

    @Brandon,
    "the adding intended has to be adding in the strict mathematical sense."

    So for the argument to get off the ground we need "adding in the strict mathematical sense" to have one and only one meaning, and for a person actually to be able to think about it without along the way hitting some other things.

    This is a considerable modification of my previous approach, which is that human beings fail to add, therefore they fail to think about addition. Whether we can actually accomplish the algorithm is neither here nor there. Ross believes I can think about an algorithm in a way that fixes it exactly and I don't think I can.

    The proposition is:
    The phrase "adding in the strict mathematical sense" refers to one and only one thing, fixing it exactly. It excludes incompossible more particular forms which might answer to the name "adding in the strict mathematical sense".

    Now I think that proposition is more problematic than we are giving it credit for.

    I would go so far as to say I think it might be false.

    ReplyDelete
  40. zmikecuber

    My level of compliance is in showing certain unsound criticism of the 1st way are bogus.

    Like misunderstanding "motion" to mean literal inertia movement from point A to B.

    As opposed to "motion" meaning change or potency being reduced to actuality.

    As for the "Burden of proof" without the underlying metaphysics worked out that is a problem.

    The 1st way is not an empirical argument in the ordinary modern sense. It's a demonstration that presupposes some form of realism.

    You would first have to argue for realism vs conceptionalism & or nominalism then moderate realism over strong realism.

    I don't know if I am compotent to do that yet.

    ReplyDelete
  41. I would go so far as to say I think it might be false.

    Perhaps, but it's hard to say what the significance of this thought is without seeing your actual reason for it.

    So for the argument to get off the ground we need "adding in the strict mathematical sense" to have one and only one meaning, and for a person actually to be able to think about it without along the way hitting some other things.

    Actually, no. It merely requires that it have identifiable meanings. In other words, the argument is not about what meanings the phrase "adding" has; likewise, there could be a jillion different meanings of "adding in the strict mathematical sense" in different contexts and it wouldn't change the argument in any direction. Your prior argument overlooked the fact that identifying something as a mistake requires already having a goal from which the mistake is a divergence; this does not change, regardless of what meaning is in view.

    It also seems a mistake to put the matter in terms of "[not] along the way hitting some other things"; it's not a matter of how many things one 'hits' but the way in which one does so.

    ReplyDelete
  42. If that is the argument then there seem to me to a couple of things wrong with it. The thing that I am bothered about just now is the idea that we have, when thinking about adding, really done the trick of fixing one and only one thing.

    What does it matter if you've fixed one and only one thing? The point is that a human being, when adding, can include some meanings and exclude others, and this disjunction cannot be accounted for in terms of the physics of the situation. The distinction between adding and quusing is real and something a human being can articuluate, even if adding is something vaguely defined - it only has to be defined clearly enough to exclude quusing to make the argument work.

    ReplyDelete
  43. What does it matter if you've fixed one and only one thing?

    How could it be trivial to the argument to admit there are multiple meanings? You are denying the premise that all formal thinking is determinate if you allow it to be just an approximation that excludes some things.

    ReplyDelete
  44. Step2,

    It's a straightforward equivocation to conflate the question of whether one and only one thing is fixed with the question of whether the fixing is approximate. Indeed, the conflation doesn't make any sense whatsoever: we can determinately fix on many things at once, without doing so by approximation; and when we are approximating, nothing about approximation itself requires that we are 'fixing' on many things at once. The two are obviously completely different questions.

    ReplyDelete
  45. If it can't be uniquely determined, and in your own words is open to a jillion contexts but is still exclusive of other things, what word or phrase would you suggest? I doubt you will claim we can fix a jillion meanings at once.

    ReplyDelete
  46. I am a newbie in learning about the philosophy of the mind so I am having difficulty following this thread. I have a very basic question regarding what is meant when it is said that formal thinking is determinate whereas physical objects are always in indeterminate. Let me ask whether this thought experiment captures the concepts.

    Consider a Chinese man who performing operations on an abacus by following instructions written in Chinese. He pushes up two beads and then pushes up two more.

    I cannot tell what he has done by observing the abacus and examining the physical characteristics of the ink and paper that contains the instructions.

    For all I know, the Chinese man added 2 + 2 or 20 + 20, or 2 million + 2 million. It is also possible that he multiplied these numbers. It is also possible that he implemented an algorithm such as a=2; if a =2 then add 2 , if not subtract 2. I cannot tell by examing the physical characteristics of the abacus, and the physical characeristics of the ink and paper that contais the instructions in Chinese. They are indeterminate as to me as the physical chacteristics of these objects have no intrisic meaning.

    The Chinese man knows what he did because his mind assigned a determinate meaning to the symbols I.e the abacus beads and the written Chinese symbols. The determinate meaning is in the mind of the Chinese man not in the physical objects themselves.

    Am I on track? or missing the point entirely??

    ReplyDelete
  47. If it can't be uniquely determined, and in your own words is open to a jillion contexts but is still exclusive of other things, what word or phrase would you suggest? I doubt you will claim we can fix a jillion meanings at once.

    I have no idea whatsoever what you are asking. 'Uniquely determining X' does not generally mean the same as 'fixing on one and only one thing'; you can uniquely determine groups or shared properties, for instance. And we obviously do this in many cases. Moreover, I never even said we couldn't ever fix things as one and only one thing; only that it was not relevant to the argument whether we could, since the argument is about the way in which 'fix on' things, not how many things we 'fix on'. So you'll have to clarify if you want an answer.

    ReplyDelete
  48. 'Uniquely determining X' does not generally mean the same as 'fixing on one and only one thing'; you can uniquely determine groups or shared properties, for instance.

    I'm using it in the sense of a unique, univocal meaning for X, whether X is a thing, group, or shared property. You interjected yourself into my response to David's claim that addition only needs to be "vaguely defined" enough to exclude something sufficiently different.

    ...since the argument is about the way in which 'fix on' things, not how many things we 'fix on'.

    We fixate on things by focusing our attention on that thing to the exclusion of other things. Every other animal is capable of hunting by this method, but for some unknown reason it is supposedly impossible for us to do the same thing with symbols.

    ReplyDelete
  49. @pauld:

    Yes, you've correctly understood the gist of the argument and what it's intended to show.

    ReplyDelete
  50. Step2,

    You interjected yourself into the discussion with reighley, to whom David T was responding; I simply don't see that your appeal to 'unique, univocal meanings' has anything to do with that discussion.

    We fixate on things by focusing our attention on that thing to the exclusion of other things. Every other animal is capable of hunting by this method, but for some unknown reason it is supposedly impossible for us to do the same thing with symbols.

    No one has said anything of the sort, so if this is your interpretation of the discussion, it's a pretty clear sign that you've simply misunderstood something. Indeed, the argument under discussion implies exactly the opposite; nor is it relevant to the discussion with reighley about whether we are fixing on one and only one thing in doing so, since exclusion is consistent with precisely defining and uniquely determining a group of things.

    ReplyDelete
  51. @Step2, Brandon, and David T:

    I'm going to borrow an example from NiV but put it to the opposite use. When I add two numbers on a calculator, I may be doing so in order to perform a multiplication using logarithms. In that sense, the calculator is both adding and multiplying (in the usual derivative sense). But it's doing both of those things quite determinately, and there's no "approximation" involved. And there are infinitely many things it's not doing: dividing by zero, baking a cheesecake, or (most relevantly here) "quadding."

    Also, a computer can at my command perform, quite unequivocally, an operation that I don't clearly understand myself, as when a mathematics student uses software to compute an integral without fully understanding integration.

    Neither of these cases involves any approximation or indeterminacy, but each involves some possibility of "multiple determination" by the user—the first because the user is actually doing more than one thing, and the second because more than one operation is (probably) consistent with the user's own limited understanding of what s/he's doing.

    I don't say those cases exhaust the possibilities, or even that there may not be others that are more on point; they're just what comes to mind offhand. But I think they do illustrate that consistency with multiple operations as determined by the user's intentions isn't the same thing as indeterminacy.

    ReplyDelete
  52. BenYachov,

    Thanks very much for your reply. I appreciate it.

    I've read The Last Superstition, as well as Aquinas, am a frequent follower of this blog, and have read certain selections of SCG and Aristotle's physics and metaphysics. I can assure you that I won't be making the stock and stupid objections which are commonly tossed around. I have two or three objections to the argument, and I'll probably stick to just those.

    We could assume the sort of realism which the argument is based upon. It would seem that the most pertinent assumption would be between act and potency.

    Anyways, I'm just looking for someone who can put up a good case, and would enjoy a casual debate. If you know anyone who matches that description, I would appreciate if you would send them my way. My username on debate.org is "zmikecuber."

    ReplyDelete
  53. A. If I may, I'd like to point out there is a lot of trouble here with the word "determinate". This is true of the discussions of several recent articles. The word seems to involve (to some):

    1. Something which is fixedly caused, as in "determinism". This has been thrashed out fairly well, but still seems to pop up.

    2. The sense of "determine" as in "find out" or "discover". This brings people to the notion that the objection is epistemological rather than metaphysical.

    3. Now, reighly and Step2 seem to take it as "I'm using it in the sense of a unique, univocal meaning for X, whether X is a thing, group, or shared property".

    B. Now, to me, the use here of "determinate" was unfamiliar; it wasn't common coin back in the day when I took philosophy, nor was it used in the older books I was reading until recently. I can't say I'm really comfortable with it, just because of the confusion.

    That, and the fact that, to me, it doesn't make clear enough the real point. Which, as I've always understood it, is that reasoning is, irreducibly, about concepts, and those concepts are related logically. OTOH, physical processes simply are not logically related. When I think confusedly, because I'm drunk, or have gotten hit on the head, the laws of physics haven't been suspended within my skull. My brain cells are not now working to new rules. No, everything is running along (physically) just as always. Analysis of my brain events is just irrelevant to the question of whether my reasoning is valid. It involves a kind of category mistake.

    While I'm quite sure of the truth of A, I ask if, in B, I have gotten confused.

    Thanks.

    ReplyDelete
  54. pauld, I think you have basically got it.

    Scott, your examples are appropriate. I was also thinking of the regular use engineers make of "vague math" because often you only need the ballpark of the answer rather than a specific number. How much will the gas cost to drive from Boston to Washington? It's 442 miles to Washington, I get 33 miles per gallon and gas costs $3.73/gallon. To make the estimation in my head easier I'll make the numbers easier - 450 miles/30 mpg = 15 gals * $3.50 per gallon is about $50. Now someone else could have done it differently and come up with a different number, but if he did it reasonably we would come up with roughly similar results. This is a formally determinate process ("estimate the gas cost") that is vaguely defined.

    ReplyDelete
  55. Indeed, the argument under discussion implies exactly the opposite; (...) since exclusion is consistent with precisely defining and uniquely determining a group of things.

    How can it imply exactly the opposite that merely physical animals can exclude other activities and desires and go around obstacles in order to pursue a goal if that is precisely what is important when we do it?

    When I think confusedly, because I'm drunk, or have gotten hit on the head, the laws of physics haven't been suspended within my skull.

    The laws of chemistry say there should be an effect on many brain functions because of excess alcohol and that is what happens.

    This is a formally determinate process ("estimate the gas cost") that is vaguely defined.

    An estimate can reasonably be called an approximation because that is part of its definition.

    ReplyDelete
  56. @Step2:

    "An estimate can reasonably be called an approximation because that is part of its definition."

    More equivocation. The process of estimation/approximation is itself determinate even if the result of the process isn't intended to be exact or precise.

    ReplyDelete
  57. @Brandon,
    "Your prior argument overlooked the fact that identifying something as a mistake requires already having a goal from which the mistake is a divergence; this does not change, regardless of what meaning is in view."

    You've misunderstood my prior argument, I don't need to be able to identify which actions are mistakes only postulate that mistakes exist without needing to know which they are. Anyway, lets leave aside my prior argument : it obviously has no legs.

    "Actually, no. It merely requires that it have identifiable meanings. In other words, the argument is not about what meanings the phrase "adding" has; likewise, there could be a jillion different meanings of "adding in the strict mathematical sense" in different contexts and it wouldn't change the argument in any direction"

    My exposition was bad. What I meant was that the argument requires that when thoughts of adding pass through my head they are actually directed at something specific and not directed at an infinite class of mutually exclusive things. My thoughts must have that feature which physical systems are supposed not to have : they must "exclude incompossible equally most particular forms".

    "'Uniquely determining X' does not generally mean the same as 'fixing on one and only one thing'; you can uniquely determine groups or shared properties, for instance."

    I think that if we let our ontology do that then physical systems stop being indeterminate too. It would then be sufficient simply to be able to exclude conclusively one possible meaning. In doing so I would be able to fix the infinite class of things that it was not.

    @pauld
    "I cannot tell by examing the physical characteristics of the abacus, and the physical characeristics of the ink and paper that contais the instructions in Chinese. They are indeterminate as to me as the physical chacteristics of these objects have no intrisic meaning."

    The position I take is that you are begging the question if you leave the Chinese man off that list. The system under consideration is abacus, ink, paper, person. You only mentioned the first three.

    @George LeSauvage.

    "A. If I may, I'd like to point out there is a lot of trouble here with the word "determinate"."

    I agree.

    "1. Something which is fixedly caused, as in "determinism". This has been thrashed out fairly well, but still seems to pop up."

    That is definitely not what Ross meant, or so I am told by reliable sources.

    "2. The sense of "determine" as in "find out" or "discover". This brings people to the notion that the objection is epistemological rather than metaphysical."

    There certainly does seem to be a lot of talk of what can be or cannot be determined from some set of variables. I think that is an interesting point though, since all the talk of physical systems and their properties starts us off on an empirical foot. Somehow we have to pull a metaphysical rabbit out of the epistemological hat.

    I think that furnishes yet another objection to Ross and Kripke both, but it doesn't happen to be the one I am pursuing. Anyway neither of your definitions fit the one Ross is using which seems to be "of a definite abstract form".

    My objection is that in fact our thoughts are not so. The objection has two branches :
    (1) that even if my thinking were _about_ something with a definite abstract form, my thinking itself is rather fuzzy (as you have no doubt already noticed). Ross needs my thinking itself to be determinate, at least some of the time, not just its object.
    (2) in fact the proposition that a formalism can be determinate in the sense Ross needs it to be is subject to some doubt.

    At the moment I wish to focus on the second branch. What does the definition of "determinate" need to be in order for Rosses argument to function?

    ReplyDelete
  58. Step2,

    Do you even read your own comments? You say:

    How can it imply exactly the opposite that merely physical animals can exclude other activities and desires and go around obstacles in order to pursue a goal if that is precisely what is important when we do it?

    This is not even coherent in the context of your original comment. I quote it again, so you can have it handy:

    We fixate on things by focusing our attention on that thing to the exclusion of other things. Every other animal is capable of hunting by this method, but for some unknown reason it is supposedly impossible for us to do the same thing with symbols.

    Now, if someone responds to this as a block something like, "No one has said anything of the sort; in fact, the exact opposite is implied by the argument under discussion," what literate person would not recognize that the only thing that this could be referring to is what is attributed to the argument under discussion? Who in this discussion even talked about what hunting animals do, except you? No one has claimed that it is impossible for human beings to fix attention by exclusion in matters of symbols; so bringing it up was, like practically everything you say, a complete irrelevance at best, and in this case, an obvious false representation of the state of the discussion since, no one on either side has suggested that they can't.

    And let's compare what I actually said with what you claimed I said. I said:

    Indeed, the argument under discussion implies exactly the opposite; nor is it relevant to the discussion with reighley about whether we are fixing on one and only one thing in doing so, since exclusion is consistent with precisely defining and uniquely determining a group of things.

    And you quoted this as:

    Indeed, the argument under discussion implies exactly the opposite; (...) since exclusion is consistent with precisely defining and uniquely determining a group of things.

    There are a few things missing there, aren't there: the part of the sentence you've kept at the front is not in fact connected to the part you left in at the end, because the "nor is it relevant" clause, which is left out, can only be read as raising a distinct point.

    Such shenanigans, if deliberate, or failure to read, if not, is not conducive to any form rational discussion. If you have something serious to contribute, contribute it; otherwise stop wasting everyone else's time.

    ReplyDelete
  59. You've misunderstood my prior argument, I don't need to be able to identify which actions are mistakes only postulate that mistakes exist without needing to know which they are.

    I didn't say you needed to identify which actions are mistakes; I said you can only identify something as a mistake by doing so as deviation with respect to a standard. Thus I did not misunderstand your argument: you are explicitly identifying as mistakes actions that you've postulated to exist, even if you are not getting any more explicit about their features than that. This is exactly the error I noted.

    I think that if we let our ontology do that then physical systems stop being indeterminate too.

    And, again, it's difficult to determine the significance of that thought without knowing your actual reasons for thinking it.

    ReplyDelete
  60. @Brandon,

    "And, again, it's difficult to determine the significance of that thought without knowing your actual reasons for thinking it."

    If I use the word "determinate" even when the object of my determination is an infinite class of things.

    Then a physical system could be reasonably said to determine the whole set of operations which it might instantiate.

    ReplyDelete
  61. "A. If I may, I'd like to point out there is a lot of trouble here with the word "determinate"."

    Feser explained it a couple of posts ago:

    "Now, what exactly is it that Δ is a symbol of? [...] There’s nothing in the physical properties of Δ that entails any of these interpretations, or any other for that matter. The physical properties are “indeterminate” in the sense that they don’t fix one particular meaning rather than another."

    ReplyDelete
  62. zmikecuber

    I may take you up on that in the future.

    No promises but it's possible.

    Cheers.

    ReplyDelete
  63. @Step2:

    'When I think confusedly, because I'm drunk, or have gotten hit on the head, the laws of physics haven't been suspended within my skull.

    The laws of chemistry say there should be an effect on many brain functions because of excess alcohol and that is what happens.'

    What you say is true (well, with qualification), but irrelevant. The physical events in my brain simply have no bearing on the question of whether my thoughts are rational or irrational. To think so is simply to miss the point.

    They may, of course, act as a kind of clue that my thoughts are likely to be irrational. Just as a pitcher may signal his pitches, or as people have "tells" which skilled observers can read as showing they are lying. But the analysis of the rightness or wrongness of the argument itself cannot be reduced to these, any more than a slider can be reduced to a twitch in the glove hand. To do so is just to miss the point.

    This is a physicalist analogue to the Freudian error of looking at the subject's wishes and motives, rather than the truth value of what he is saying.

    ReplyDelete
  64. If I use the word "determinate" even when the object of my determination is an infinite class of things.

    Then a physical system could be reasonably said to determine the whole set of operations which it might instantiate.


    But this doesn't follow; at the very least you need a reason motivating the inference, which is precisely my point. You haven't, for instance, established that the determination in the conclusion actually is a 'reasonable' extension of the determination in the premise, which is precisely one of the points that would be in dispute; and you would need an account of how physical systems instantiate operations that would not, in fact, concede the whole point to Ross and Feser, since one way to understand what it means for a physical system to instantiate operations is for it to do so by external interpretation.

    ReplyDelete
  65. Brandon,
    I apologize for misreading your comment, I do not apologize for making points you think are irrelevant. If you don’t want to waste your time then don’t respond to my comments, it should not be difficult since you are convinced they are completely irrelevant and off-topic. As for everyone else, they can judge for themselves how they spend or waste their time.

    Scott,
    The same thing can be said about quaddition. I mean the process of quaddition determines a result, it just isn't precise.

    George,
    1. The physical events in my brain simply have no bearing on the question of whether my thoughts are rational or irrational. 2. They may, of course, act as a kind of clue that my thoughts are likely to be irrational.

    I'm not sure how you reconcile those statements. Alcohol intoxication cannot have “no bearing on rationality” and also be a true indicator of the likelihood of irrationality. If heaven forbid anyone you know develops Alzheimer’s there is no reason to believe their cognitive dysfunction is caused by anything other than brain deterioration.

    ReplyDelete
  66. " and you would need an account of how physical systems instantiate operations that would not, in fact, concede the whole point to Ross and Feser, since one way to understand what it means for a physical system to instantiate operations is for it to do so by external interpretation. "

    Ross's argument seems to me to be something like this.

    I propose to Ross that a physical system as A Definite Abstract Form X.

    Ross objects that no matter what X is, the physical system is logically consistent with an infinite number of incompossible equally most particular forms.

    I answer Ross that my choice of the X was an infinite set of functions, the set of meanings that might have been ascribed to X in a logically consistent way.

    Which is to say that, when presented with a calculator, the property "behaves as this calculator" could be taken to be a definite abstact form, and if it could be so taken then Ross's argument makes no sense. It is not in fact the case that there are any forms incompossible with "behaves as this calculator" which are also equally most specific to this calculator. The calculator behaves as it does. Tautology full stop.

    So why does Ross exclude "behaves as this calculator" from the realm of abstract thought?

    My feeling is that he finds that predicate to be too broad. He would like to force me to be more specific about what it means "to behave as this calculator", perhaps by some ill advised mapping of the physical states of the system onto the integers. To convince me to do this he needs his ontology to do some of the heavy lifting for him. It must be, in Ross's world, that not all definitions are definite enough.

    ReplyDelete
  67. "D. Machines are purely physical.

    No physicalist will accept D unless it's qualified with D2a: Humans are purely physical, or D2b: Humans are machines. And that makes the rest of the proof pointless.

    ReplyDelete
  68. But then you throw truth preservation out the window.

    ReplyDelete
  69. I propose to Ross that a physical system as A Definite Abstract Form X.

    This is an interesting idea, and you are right that it is, among various options, probably the most reasonable platform for response, but it runs into the question of what work 'abstract' is doing here. If 'abstract' means that it is a definite form as abstracted by a mind, which is the most natural way of talking about things being abstract, then it would concede the entire ground to Ross (in addition to requiring, if all physicals systems are definite abstract forms, that idealism be true). Ross has no issues with claiming that mental abstractions can be determinate; that's the whole point. So it has to have some other meaning.

    ReplyDelete
  70. reighley,

    "Which is to say that, when presented with a calculator, the property "behaves as this calculator" could be taken to be a definite abstact form, and if it could be so taken then Ross's argument makes no sense."

    In Ross's terms, that doesn't count as a "physical process".

    You can think of the physics of small systems in state-machine terms, which requires a set of states, and a set of transition functions that specify successor states at later times given an initial state. The phrase "behaves as this calculator" is talking about the transition function.

    But to Ross, a physical process is *the list of successive states*. The idea of there being a definite transition function inherent in the mechanism is not included. And no list of successive calculator states can ever be enough to pin down the precise transition function it is operating by.

    He does briefly address the question in the section starting "What happened to Nature?" His objections being that mathematics is an idealisation of the physics, and that the history of states physics goes through is compatible with other laws of nature, too.

    I'm guessing his reason for doing this - treating physical processes as no more than a history of successive states - is that transition functions would involve counterfactuals; things that could have happened but didn't. I expect this touches on the AT actuality-potentiality business. But I don't know for sure, and he isn't clear.

    Anyway, the possibility of a defined transition rule is excluded from physical processes by his definition of physical process, and included in human reasoning by a sort of argument from adverse consequences - that we couldn't reason even *this* far if there wasn't a definite truth-preserving transition rule by which logic could operate. It's not credible to us that there aren't such rules.

    I can't say I agree, but there may be more to the argument that I'm not seeing.

    ReplyDelete
  71. Step2,

    George,
    1. The physical events in my brain simply have no bearing on the question of whether my thoughts are rational or irrational. 2. They may, of course, act as a kind of clue that my thoughts are likely to be irrational.

    I'm not sure how you reconcile those statements. Alcohol intoxication cannot have “no bearing on rationality” and also be a true indicator of the likelihood of irrationality.


    There isn't anything to be reconciled.

    George's 2nd statement deals with thoughts in general ('likely to be'), and his 1st statement with thoughts in particular ([in fact] 'are').

    IOW, to say that the thoughts of an intoxicated person are likely to be irrational is not to say that the particular thoughts he is thinking are irrational.

    Example: if an intoxicated person leaves a bar and notices that the ground is wet, that he is intoxicated has no bearing on whether his subsequent thought "it must have been raining" is rational or irrational.

    ReplyDelete
  72. @Step2:

    "George,
    1. The physical events in my brain simply have no bearing on the question of whether my thoughts are rational or irrational. 2. They may, of course, act as a kind of clue that my thoughts are likely to be irrational.

    I'm not sure how you reconcile those statements. Alcohol intoxication cannot have “no bearing on rationality” and also be a true indicator of the likelihood of irrationality. If heaven forbid anyone you know develops Alzheimer’s there is no reason to believe their cognitive dysfunction is caused by anything other than brain deterioration."

    To expand on what Glen is saying, the error I think you are making is a categorical one, confusing grounds for a conclusion with causes of events.

    If someone says something senseless, and we explain it by drugs or brain disfunction, that is causal, why he speaks rot. But it doesn't at all address the question of why the statement is senseless. That can only be determined by analyzing the statement logically. And the difference between logical entailment (or conformity with the facts, or conceptual coherence) is different in kind than that between cause and effect.

    This is obscured for 3 reasons:

    1. We do use "cause" equivocal senses, as the "causes of WWI", which are not efficient causes at all. That doesn't change the fact that physical causation is different from logical entailment (etc.)

    2. As a practical fact, we do tend to ignore what drunks say. But that is just a shortcut. Sometimes, as Glen points out, they still make sense. Had we but world enough and time we should always work out the truth of what they say, but we don't. The case is analogous to dealing with a known liar. We ignore him, although sometimes he may just be telling the truth.

    3. This confusion is a very popular fallacious rhetorical tactic, and not always noticed. Ad hominem arguments are not bad because they are mean and unkind, but because they change the subject. Freudians and Marxists are notably prone to this gambit, as it's built into the system, but all political and religious arguments are in danger of falling into it. It's well to keep in mind Chillingworth's example: If it were proved that St Thomas were secretly a Moslem or a Jew, the arguments in the Summas would still need to be addressed as they stand. (After all, we cannot discount a legal brief just because we know the lawyer was paid to write it for one side.)

    Motives and causes are on one side; reasons and grounds on another; they are categorically different.

    ReplyDelete
  73. donjindra,

    You don't seem to understand how arguments work. Premises are to be accepted if they are argued for convincingly, or are self-evident. It is not important whether or not a preconceived perspective would accept thme.

    Physicalists may not psychologically want to accept D, but this is irrelevant. What is relevant is whether or not proof is given for D. If so the physicalist must evaluate the proof. You appear to be confused.

    ReplyDelete
  74. @Brandon,
    "If 'abstract' means that it is a definite form as abstracted by a mind, which is the most natural way of talking about things being abstract, then it would concede the entire ground to Ross (in addition to requiring, if all physicals systems are definite abstract forms, that idealism be true)."

    I don't pretend to be able to parse completely the term "definite abstract form". I am just parroting Ross so that I don't once again get into the trouble of inserting my own terms of art in place of his.

    The point is that whatever he means, he intends it to be a property that a physical system does not have and that our thoughts do have. So he must not have used "abstract" to require thoughts by definition or he would be deliberately begging the question. It must be possible, at least in theory, for things other than thoughts to take on definite abstract forms. His goal is to show that physical systems are not such things, and therefore our thoughts are physical systems.

    @NiV
    "But to Ross, a physical process is *the list of successive states*. The idea of there being a definite transition function inherent in the mechanism is not included."

    If that is the case then I am inclined to agree with Ross that thoughts are not such things. It's a bit of a straw man though since I'm not sure anybody actually holds this view of physical processes (ie as being without a transition function). A great deal of ink has been spilled on what that transition function might be but I think darn near everybody agrees that there must be one.

    ReplyDelete
  75. "Premises are to be accepted if they are argued for convincingly, or are self-evident. It is not important whether or not a preconceived perspective would accept them."

    1) There is no way I see that the premises can be argued for successfully without first accepting mutually exclusive categories which effectively beg the question.

    2: The premises are certainly not self-evident.

    Contrary to your assertion, it's crucial that preconceptions permit the acceptance of all premises in a proof.

    ReplyDelete
  76. You'll have to explain one your first point again to me.

    In the comments post of Oeter's post that Feser is here replying to you tried to make the same point I think you are now: that no materialist would accept premise 2, or B, of Feser/Ross argument and therefore it is question begging. Which categories are you talking about? You seem to be trying to shift your original argument, as well. At the moment you look, as often happens, extremely confused by the whole debate, so don't be surpised that I do not take your concerns as convincing.

    This would be true if this premise were simply being asserted. However, it is not. Both Feser and Ross argue for the premise. You simply do not seem to understand how rational argument works.

    ReplyDelete
  77. You'll have to explain one your first point again to me. Which categories are you talking about? You seem to be trying to shift your original argument, as well. At the moment you look, as often happens, extremely confused by the whole debate, so don't be surpised that I do not take your concerns as convincing.

    In the comments post of Oerter's post that Feser is here replying to you tried to make the same point I think you are now: that no materialist would accept premise 2, or B, of Feser/Ross argument and therefore it is question begging.

    This would be true if this premise were simply being asserted. However, it is not. Both Feser and Ross argue for the premise. You simply do not seem to understand how rational argument works.

    ReplyDelete
  78. "Which categories are you talking about?"

    He means that the premises require you to accept that 'machines' and 'humans' are mutually exclusive categories.

    ReplyDelete
  79. A. All formal thinking is determinate.

    B. No physical process is determinate.

    C. No formal thinking is a physical process. [From A and B]

    D. Machines are purely physical.

    E. Machines do not engage in formal thinking. [From C and D]

    F. Humans are machines.

    G. Humans do not engage in formal thinking. [From E and F]

    I think that is what donjindra is talking about.

    ReplyDelete
  80. @FZ:

    Could be. However, the order of the actual argument is:

    A. All formal thinking is determinate.

    B. No physical process is determinate. (Proof: If it were, then physical processes—those in machines, for example—would instantiate pure functions. But they do not, because any physical process is consistent with multiple incompossible pure functions.)

    C. No formal thinking is a physical process. [From A and B]

    D. Humans engage in formal thinking. (This premise is itself supported by argument.)

    E. Human thought is not purely physical. [From C and D]

    ReplyDelete
  81. (The following comment on B—which is not part of the proof—should be added: At most, machines instantiate pure functions in a derivative or analogical way, strictly dependent on human intentions.)

    ReplyDelete
  82. If one reads donjindra's comments on Oerter's post being discussed in this thread or his recent contributions here one finds he seems to not understand how rational arguments work, or at least he misunderstands what the concept of begging the question means.

    For example:

    All that and you still haven't explained why I as a materialist or an agnostic on the issue would ever agree to premise (2). Fact is, I'm forced to agree to the conclusion prior to the proof. That's begging the question.

    I don't deny I'm begging the question by sticking to -q. But I'm not trying to prove -q.

    Here's a similarly structured "proof":

    All humans have free will
    No clump of matter has free will
    Thus humans are not clumps of matter

    But this, like Ross's proof, proves nothing.


    He seems to not understand that Ross and Feser argue for (2) and therefore it is not question begging.

    In this instance, whether or not there is an important distinction between men and machines is a consequence of the argument. It is not relevant whether or not the materialist is unlikely to accept this before they even grapple with the argument.

    As I have pointed out before, he conflates different levels of analysis of arguments: validity, the truth value of premises, and psychological responses to argument.


    ReplyDelete
  83. I notice, since I first read Oerter's combox, the discussion has continued and donjindra has started talking about mutually exclusive categories, or false dichotomies. I wouldn't like to speculate on the view that he is just making things up ad hoc. The important point is he makes his basic point again:

    For a physicalist, Ross's P2 ignores the fact that physical processes include all thinking of any sort. And you could claim this begs the question too. And that's fine. But that doesn't give Ross the authority to beg the question from the other direction.

    This is patent nonsense. Ross does not ignore this claim: whether or not physical processes can account for formal thinking is what the argument is about. It would only be question begging if premise (2) were not being argued for my Ross, Feser, or anyone making the argument; but it is being argued for.

    ReplyDelete
  84. FZ,

    Yes, I think that's what he was suggesting.

    Scott,

    You can do that, so long as you add:

    F Humans are machines.

    G Some machines are not purely physical. [From E and F.]

    Anonymous,

    "He seems to not understand that Ross and Feser argue for (2) and therefore it is not question begging."

    If they argue for 2, then you need to include the premises and steps of that sub-argument in the ABC... outline.

    ReplyDelete
  85. There is no necessity of adding all the arguments Ross and Feser and others make to support (2) in the outline. It depends upon the use being made of outline. Feser composed it, I believe, to simply outline the primary premises and the conclusion of the argument. In the context, it seems an appropriate outline, given the background offered by Feser in the exchange. That is one of the points: it is as if donjindra doesn't really follow the discussions.

    In fact, if memory serves, wasn't this why he was banned from posting here?

    ReplyDelete
  86. The problem with DJ is that he absolutely dislikes Natural Law, period. It lends credence to ideas he finds threatening, so it has to be wrong.

    Notice that DJ almost never asks a question, or looks for a clarification. It's just again and again, "Your argument is wrong, period, and it's totally refuted, because of what I just said." Then someone points out the flaws in what he just said, and why his reasoning doesn't work. Without missing a beat, he changes to a brand new reason why the argument is wrong, period, totally refuted. Repeat until the comments start getting removed and the ban is enforced.

    It's boring.

    ReplyDelete
  87. @Scott,

    "B. No physical process is determinate. (Proof: If it were, then physical processes—those in machines, for example—would instantiate pure functions. But they do not, because any physical process is consistent with multiple incompossible pure functions.)"

    Let's suppose for the moment that I thought that this was a valid deduction.

    In order to carry the rest of the proof through you would need a similar lemma attached to A that convinced me that formal thinking _is_ determinate in the sense Ross needs it to be.

    I believe that Ross's defense is that we cannot claim formal thinking to be indeterminate without being incoherent, since the statement "formal thinking is not determinate" is an example of determinate formal thinking. I do not think this is adequate to the task that it has here been assigned. Mainly because I believe the thought "formal thinking is not determinate" to be consistent with multiple, mutually exclusive interpretations of the terms.

    ReplyDelete
  88. Also, NiV, donjindra's point was about physicalists not accepting that humans are not machines or purely physical. This is quite different to your E and F. His point is aimed right at the basis of the discussion and is based on the irrelevant view that a physicalist will not accept a view that contradicts physicalism.

    Another Anon,

    Indeed, although I'd add he is not just opposed to natural law, but he seems to have a hard time comprehending the arguments and following the discussions - although the latter may have a lot to do with choice.

    Step2 has a similar modus operandi.

    ReplyDelete
  89. "Mainly because I believe the thought "formal thinking is not determinate" to be consistent with multiple, mutually exclusive interpretations of the terms."

    I'm not sure I follow here. Can't one specify and clarify the terms instead of "meaning" a bunch of incompatible things?

    ReplyDelete
  90. "It depends upon the use being made of outline. Feser composed it, I believe, to simply outline the primary premises and the conclusion of the argument."

    Possibly there is some confusion over what is premise and what is (intermediate) conclusion?

    The premises to an argument are the statements accepted initially without proof. You set out the premises, and then use syllogisms to combine them until you have your conclusion. That's what donjindra seems to be assuming you're doing.

    But if as you say these are not premises but intermediate conclusions from parts of the argument not shown, then it's not a complete outline of the argument, and there is potentially more that a critic could object to.

    As you say, it depends on what was meant by the term 'outline'. Which terms are actually premises?

    ReplyDelete
  91. If that is true, why all the talk about question begging? The argument, as presented in its standardised form, is not question begging.

    ReplyDelete
  92. @reighley:

    "In order to carry the rest of the proof through you would need a similar lemma attached to A that convinced me that formal thinking _is_ determinate in the sense Ross needs it to be."

    Fair enough, but whether or not I included such a lemma in my summary, I think we do know that when we add, we're just plain adding and not doing anything else. As Ross himself argues, even to suspect that we might be doing otherwise still presumes that we know what addition is.

    ReplyDelete
  93. @reighley:

    "I believe that Ross's defense is that we cannot claim formal thinking to be indeterminate without being incoherent, since the statement "formal thinking is not determinate" is an example of determinate formal thinking. I do not think this is adequate to the task that it has here been assigned. Mainly because I believe the thought "formal thinking is not determinate" to be consistent with multiple, mutually exclusive interpretations of the terms."

    From J. Ross, "Immaterial aspects of thought", pdf pg. 3:

    "Can judgments really be of such definite "pure" forms? They have to be; otherwise, they will fail to have the features we attribute to them and upon which the truth of certain judgments about validity, inconsistency, and truth depend; for instance, they have to exclude incompossible forms or they would lack the very features we take to be definitive of their sorts: e.g., conjunction, disjunction, syllogistic, modus ponens, etc. The single case of thinking has to be of an abstract "form" (a "pure" function) that is not indeterminate among incompossible ones."

    The very next paragraph:

    "The same point again. I can reason in the form, modus ponens ("If p then q"; "p"; "therefore, q"). Reasoning by modus ponens requires that no incompossible form also be "realized" (in the same sense) by what I have done. Reasoning in that form is thinking in a way that is truth-preserving for all cases that realize the form. What is done cannot, therefore, be indeterminate among structures, some of which are not truth preserving[6]. That is why valid reasoning cannot be only an approximation of the form, but must be of the form. Otherwise, it will as much fail to be truth-preserving for all relevant cases as it succeeds; and thus the whole point of validity will be lost. Thus, we already know that the evasion, "We do not really conjoin, add, or do modus ponens but only simulate them," cannot be correct."

    And the footnote (just to preempt one possible objection):

    "[6] I am not, of course, suggesting that a valid course of reasoning is not also a case of a variety of invalid forms, e.g., "P, therefore, C." But it must determinately be a case of some valid form."

    This is in the second and third page of the article. There is no need to guess, just google for "Immaterial aspects of thought" and you will hit a link to the paper.

    ReplyDelete
  94. Anonymous,

    "If that is true, why all the talk about question begging?"

    Because I think donjindra is (incorrectly) interpreting them as premises to the argument - i.e. as statements to be assumed.

    The argument has to proceed from premises so simple and self-evident that they *can't* argue, through a valid chain of reasoning to the conclusion they don't like. Because you're starting half way, with statements that are *not* self-evident, much of the meat of the argument is hidden away in the 'premises'. When premises are interpreted as unproven assumptions that are supposed to be just accepted, that would imply that a big chunk of the argument was being assumed.

    When you realise that they're not actually premises, but the conclusions of sub-arguments each with their *own* premises, the correct question to ask would be whether a physicalist would have to accept all these sub-argument premises. That's not clear, without knowing what they all are.

    ReplyDelete
  95. @reighley:

    Let me add that J. Ross gives other arguments further down the article for why denying the determinacy of though has astronomical costs and cannot be coherently maintained (which I presume is what you were trying to get at).

    ReplyDelete
  96. "They have to be; otherwise, they will fail to have the features we attribute to them and upon which the truth of certain judgments about validity, inconsistency, and truth depend;"

    Argument from adverse consequences?

    ReplyDelete
  97. @NiV:

    "Argument from adverse consequences?"

    If you are willing to bite the bullet and pay the price that we cannot think at all to stave off the inevitable conclusion, I suppose it is no biggie to call it "an adverse consequence". I certainly have no qualms about it.

    ReplyDelete
  98. @reighley:

    "Argument from adverse consequences?"

    Argument from self-stultification. If we can't even be sure our thought is instantiating the form of a valid argument, then our reasoning isn't to be trusted.

    ReplyDelete
  99. NiV,

    I think your being far too charitable to donjindra, who after all has a track record of such behaviour.

    Also, I just don't think your point is relevant given the context. Anyone who had read the discussion, and donjindra has commented on it from the start, should be able to properly judge what is going on and see just the status of overview Dr. Feser gave and the support for each subargument.

    ReplyDelete
  100. @grodrigues,
    "Reasoning in that form is thinking in a way that is truth-preserving for all cases that realize the form. What is done cannot, therefore, be indeterminate among structures, some of which are not truth preserving[6]"

    Doesn't this argument permit indeterminacy among structures all of which are truth preserving?

    So indeterminacy is not by itself enough. It needs to be indeterminacy among possible forms some of which are not truth preserving.

    @Scott,
    This makes it harder to point the finger at the machines. They do not need to instantiate pure functions _and neither do we_ in order to make valid deductions. It is sufficient that the whole class of incompossible functions the machine might instantiate be truth preserving.

    I actually think the condition on a validly reasoning machine is weaker still, since I take the issue of making mistakes which lands in that footnote to be much more serious than anyone else seems to, but I fumbled that one early on so I'll just grant it and sulk.

    ReplyDelete
  101. @reighley:

    "It is sufficient that the whole class of incompossible functions the machine might instantiate be truth preserving."

    With respect to what interpretation(s), and where does it (do they) come from?

    ReplyDelete
  102. I don't think it's being fully appreciated by some exactly how expansive the range of possible functions for any given physical state is. This isn't like a given structure can mean either 2 + 2 = 4 or perhaps 2 + 2 = 5, and that exhausts the possibilities.

    ReplyDelete
  103. @Scott,
    "With respect to what interpretation(s), and where does it (do they) come from?"

    I don't think that the answer to this question matters to the structure of Ross's argument. If we actually had to have an outside interpreter present and accounted for in order for the machine to instantiate functions even in the indeterminate way that it does then Ross really is question begging.

    My reasoning is runs like this :
    We assume (A) that a physical system cannot fix an interpretation uniquely, excluding all others.

    but it does not follow from (A) that a machine cannot do part of the work of interpretation. It might be able to exclude some possible interpretations.

    So I propose (B) that a physical system can exclude a non-empty class of interpretations. Which is to say that even though there is an infinite space of possible interpretations which some mind might ascribe to some physical system, there is also a space of interpretations which no reasoning mind would ever ascribe to that same physical systems. Certain interpretations are logically impossible.

    My justification for (B) is that it must be so or we would not be able to reason about the machine itself. So for instance it cannot be that one possible interpretation of the machine is that it does not do what in fact it does. The machine itself excludes it.

    My objection amounts to this : we have been the victims of a bait and switch. We may think we can demonstrate that (A) is true, but what we need to show is that (B) is false.

    ReplyDelete
  104. Surely, all that shows is the physical has some bearing on the meaning of the physical? I some sense thought about the physical requires input, knowledge, of the physical. That hardly seems to undermine the argument to me. The indeterminacy of the physical remains being referred to remains.

    ReplyDelete
  105. - That should be: In , not I , some sense thought about the physical....

    ReplyDelete
  106. Which is to say that even though there is an infinite space of possible interpretations which some mind might ascribe to some physical system, there is also a space of interpretations which no reasoning mind would ever ascribe to that same physical systems. Certain interpretations are logically impossible.

    Just for fun? Let's see them. And let's make sure they are logically impossible.

    ReplyDelete
  107. Jumping in here to add,

    So I propose (B) that a physical system can exclude a non-empty class of interpretations. Which is to say that even though there is an infinite space of possible interpretations which some mind might ascribe to some physical system, there is also a space of interpretations which no reasoning mind would ever ascribe to that same physical systems. Certain interpretations are logically impossible.

    My justification for (B) is that it must be so or we would not be able to reason about the machine itself. So for instance it cannot be that one possible interpretation of the machine is that it does not do what in fact it does. The machine itself excludes it.


    While I'd like to see these "logically impossible interpretations" that are made logically impossible owing to the machine itself, I think your second paragraph contains a problem.

    You say that it must be possible for a physical system to exclude a non-empty class of interpretations, because otherwise we wouldn't be able to reason about "the machine itself". But based on what you're saying, it sounds like we're not reasoning about "the machine itself" - we're reasoning about interpretations related to the machine. If that's the case, then it doesn't seem as if it's necessary for a machine to exclude a non-empty class of interpretations - it's just necessary for one particular mental interpretation to exclude them. 'The machine itself' isn't the relevant party, so to speak.

    ReplyDelete
  108. I'll also add, with Taylor, that I don't think this is really hitting Ross' point. "Okay, an infinite amount of interpretations are possible for any given physical state, including irrelevant or mutually incompatible interpretations, but..!" seems flawed from the outset, at least as a response to Ross.

    But the claim seems fun in and of itself, so hey, let's see where it goes.

    ReplyDelete
  109. reighley,

    "Doesn't this argument permit indeterminacy among structures all of which are truth preserving?"

    I don't think so, if you follow it through. It's not a constraint the argument allows to be applied.

    If you apply Ross's machine argument to human reasoning, you would get something like the following.

    Human reasoning is no more than the sequence of the outcomes that actually occur. Humans applying modus ponens up to now may have coincided with the pure function modus ponens, but that's not a guarantee that they always will, or that the actions are the same. The human might be doing modus ponens unless one of the terms is 'two blue squares', or 'a china teapot in orbit around Saturn', or until the human goes senile, or unless the human really, really doesn't like the conclusion. If you haven't explored all possible inputs, you can't know that the human's output in all cases will be correct, because there are lots of incompatible, 'almost ponens' rules that would fit the events just as well.

    For that matter, can *you* be assured that your brain will apply the rule correctly on every possible input? We know there are optical illusions and fallacies and paradoxes in which the brain's default machinery makes errors of deduction. Can you be *absolutely sure* that there are no possible inputs for which your brain wouldn't suffer a 'logical illusion' and give the wrong answer? Given that the only test permitted is to actually try it?

    Pure functions involve counterfactuals, specifying the output for *every* possible input simultaneously. But actuality is *single* (ignoring many-worlds QM for the moment) and can only test out *one* set of inputs. And the next trial is a whole new different situation.

    The additional non-physical element that Ross's argument requires humans to have is the counterfactuals defining their output in all the different circumstances that didn't actually occur. Counterfactuals don't *exist*; they're not physical, or determinable from the history of events that actually occur.

    The question is, does that argument apply with any less strength to humans? Never mind that if it does we cannot rely on our own reasoning; that's just adverse consequences. Does it apply? And if not, why not? And why does that not apply to machines following the laws of physics (which are likewise pure functions involving counterfactuals)?

    ReplyDelete
  110. @reighley:

    "Doesn't this argument permit indeterminacy among structures all of which are truth preserving?"

    The particular argument considered in isolation, maybe, but not in conjunction with the other arguments.

    "So indeterminacy is not by itself enough. It needs to be indeterminacy among possible forms some of which are not truth preserving."

    So let us assume for the sake of argument that indeterminacy can somehow be contained to be only among the truth-preserving forms. Use the Curry-Howard isomorphism between proofs and programs (this makes the illustrative examples simpler to state). What you are saying is that we would have programs which we do not, and cannot, know are correct but that would compute the correct answers; we would not know that computers work as they should. But if we were not, and could not, be able to know the programs were correct how could we ever be justified in believing that the program spitted out the correct answer? A proof is not simply a function from the set of true propositions to itself as is attested by the fact that the same theorem admits of different proofs (*). And it follows that we would *not* know the truth of certain propositions like "P is a proof of p". Proof theory to the trash bin. This particular proof in the present paragraph to the trash bin: the conclusion is right, but the proof is incorrect.

    "The last act is the greatest treason.
    To do the right deed for the wrong reason.
    ― T.S. Eliot, Murder in the Cathedral"

    But of course, we do know determinately that some programs are correct and that "P is a proof of p" for instances of P and p. We know determinately that certain programs are incorrect. We even know that the halting problem is not solvable. And we know it because we can prove it. We determinately apply modus ponens, not modus tollens or p |- p or whatever other valid deductive rule the deductive calculus has (**).

    (*) Ross goes over this point in the article: "I propose with some simple cases to reinforce the, perhaps already obvious, point that the pure function has to be wholly realized in the single case, and cannot consist in the array of "inputs and outputs" for a certain kind of thinking."

    (**) for those out there that actually know about this stuff, instead of just mindlessly parroting things like me, the same could be said about calculi which have only one deductive rule but a bunch of axioms like Hilbert calculus; it is just a matter of changing the details in the argument a little bit.

    ReplyDelete
  111. Anonymous,

    "He seems to not understand that Ross and Feser argue for (2) and therefore it is not question begging."

    I understand that. I deny this objective was accomplished and see no way it can be accomplished. Arguing for a proposition is not going to help if those arguments are made with the same sort of question-begging assumptions. That's what Ross and Feser do, one way or another. I have not changed from this position.

    Rational arguments must be based on non-question begging lines of argument. I do understand that much about rational argument. Furthermore, this "philosophy of mind" issue has nothing to do with natural law, which I don't "absolutely dislike," btw.

    So, since I've been accused of never asking questions, I'll ask why as a "physicalist" I should accept *any* line of reasoning that first assumes mutually exclusive subcategories like man/machine or pure/simulated addition? Why is my preconception to be discarded while yours is not? Why is such a demand on my foundation seen as "rational" rather than biased?


    ReplyDelete
  112. @NiV:

    "Never mind that if it does we cannot rely on our own reasoning; that's just adverse consequences."

    If you really do believe this, then you are no better than the brute animals and you should excuse yourself from the table of rational conversation.

    note: I know there are some rude, obnoxious people out there that will take this as a justification to pile invective and abuse, irony and sarcasm, on their poor hapless victims; but I sternly remind those people that already in the OT (quoting from memory) it is said that the righteous man cares for his domestic animal.

    ReplyDelete
  113. "Never mind that if it does we cannot rely on our own reasoning; that's just adverse consequences."

    If our reasoning is not reliable how does "that's just adverse consequences" follow from "Never mind that if it does we cannot rely on our own reasoning"?

    ReplyDelete
  114. Along the same lines and with a nod toward Ross's own argument, how would we know that there even is such a thing as reliable reasoning for our "brains" to fail to instantiate?

    ReplyDelete
  115. "I don't think it's being fully appreciated by some exactly how expansive the range of possible functions for any given physical state is."

    I think you're right on target about that. In fact, as I've implied between the lines a couple of times in this thread and others, I'm not at all sure what principles are being used to rule out possible "meanings" of physical states and behaviors, or even whether any such principles are possible.

    ReplyDelete
  116. grodrigues,

    "If you really do believe this, then you are no better than the brute animals and you should excuse yourself from the table of rational conversation."

    :-) If this is so, then I am no better than the machinery! I know my place!

    Anonymous,

    "If our reasoning is not reliable how does "that's just adverse consequences" follow from "Never mind that if it does we cannot rely on our own reasoning"?"

    Because it's entirely possible that our reasoning *is* unreliable, and if a proof was to reveal that, we should take it seriously.

    'Argument from adverse consequences' (the argument that it can't be true because it would imply something really bad if it was) is a fallacy. Being bad or undesirable or expensive has nothing to do with whether it is true or not.

    Consider the experience of Bertrand Russell on discovering the Russell paradox undermining the foundations of Frege's logical edifice - then probably the most precise exposition of human rationality that existed. People can and do take such claims seriously.

    But more importantly, it's important to understand the actual argument by which we show that the argument *doesn't* apply to humans (if, indeed, it doesn't) because that will tell us a lot about what this thing is that humans have and machines don't, and how we can know.

    It's not an issue for me personally, because I don't buy Ross's account of physical processes as no more than the history of states/outcomes. I think counterfactuals are implemented explicitly in the physics, and so there's no problem at all with machines implementing pure functions (to the extent that any finite, approximately-constructed machine can). But Ross's idea is an interesting hypothetical worth exploring. Suppose physicalists took the argument seriously and concluded that humans *can't* reliably reason. What then?

    ReplyDelete
  117. @NiV:

    "Suppose physicalists took the argument seriously and concluded that humans *can't* reliably reason. What then?"

    Then the argument that got them to that conclusion couldn't be trusted either. That was pretty much Anon's point; drodrigues and I have each made versions of it as well.

    ReplyDelete
  118. "Because it's entirely possible that our reasoning *is* unreliable, and if a proof was to reveal that, we should take it seriously."

    Wouldn't that also apply to any reasoning that brought us to the proof in the first place?

    ReplyDelete
  119. "Because it's entirely possible that our reasoning *is* unreliable, and if a proof was to reveal that, we should take it seriously."

    Ummm......never mind.

    ReplyDelete
  120. Quick question:

    How does one decide whether or not something is approximate, without having some idea as to what the ideal is?

    ReplyDelete
  121. Scott,

    "Then the argument that got them to that conclusion couldn't be trusted either. That was pretty much Anon's point; drodrigues and I have each made versions of it as well."

    True. But that counterargument reaches the same conclusion by a different route.

    If you can't rely on your own reasoning, you can't safely conclude anything. But that doesn't necessarily mean that you won't still believe that you can. Down this road, madness lies. :-)

    FZ,

    "How does one decide whether or not something is approximate, without having some idea as to what the ideal is?"

    Because it *almost* solves the problem that the ideal solves exactly.

    For example, we knew Newton's law of gravity was approximate long before we knew the real law (if we do), because it gave very accurate but not quite perfect predictions, and fitted a lot of the mathematical requirements but not all of them.

    ReplyDelete
  122. @NiV:

    "Because it *almost* solves the problem that the ideal solves exactly."

    That doesn't answer FZ's question, which was (emphasis added): "How does one decide whether or not something is approximate, without having some idea as to what the ideal is?"

    ReplyDelete
  123. @NiV:

    "If you can't rely on your own reasoning, you can't safely conclude anything."

    Including that. Welcome to madness.

    ReplyDelete
  124. Scott,

    "That doesn't answer FZ's question, which was (emphasis added): "How does one decide whether or not something is approximate, without having some idea as to what the ideal is?""

    Oh, I see. You mean how do you tell if 42 is the answer when you don't even know what the question is?

    The short answer is: you can't.

    ReplyDelete
  125. @NiV:

    "The short answer is: you can't."

    I agree, and I think that concedes the most important point at issue. In order to recognize that a piece of (apparent) reasoning ("Then [the dog] is a father, and he is yours; ergo, he is your father, and the puppies are your brothers") is an unsuccessful syllogism, we have to know what a syllogism is.

    ReplyDelete
  126. donjindra,

    The physicalist need not accept such mutually exclusive categories, and this argument is not asking them to accept them. Or rather, these categories are only established as a consequence of the argument. And as consequences of the conclusion it is irrelevant what the physicalist view of them is if they do not properly grapple with the argument itself.

    These mutually exclusive categories simply are not in the argument.

    ReplyDelete
  127. To put it another way: F and G in Ed's summary of Ross's argument don't depend on D and E.

    ReplyDelete
  128. Scott,

    "I agree, and I think that concedes the most important point at issue."

    If syllogisms are a case of not knowing what the problem you're trying to solve is, then yes. Are they?

    Let's say an 'approximate syllogism' was a process that when given true propositions, yielded a new proposition that had at least a 95% probability of being correct. (The usual scientific significance threshold.)

    Do we know what the ideal is? Yes, a 100% probability. Do we actually achieve the ideal? If you're using statistics, 100% is generally impossible to achieve (except in special cases). Can we reason reliably? Up to a point, so long as we don't chain together too many steps.

    Does our recognition of what the ideal case would be imply that the ideal must be physically possible or logically consistent? No, not in this case. We'd need an infinite sample size, which is physically impossible and even hypothetically would involve all sorts of logical difficulties. (Superprocesses, anyone?)

    Even showing it was impossible with less than 100% confidence would be trouble. OK, *maybe* you're wrong. But you're probably not.

    It's not the same situation as with Ross's approach, so I make no claims that the same applies. It's just a toy example to explore the idea. But the idea of having a provably unreliable form of human reasoning is not totally crazy! :-)

    ReplyDelete
  129. Oops. I meant 'supertasks', not 'superprocesses'.

    ReplyDelete
  130. @NiV:

    "If syllogisms are a case of not knowing what the problem you're trying to solve is, then yes. Are they?"

    Huh? My point (and FZ's) was that we do and must know the ideal in order to tell whether we're approximating it.

    "Let's say an 'approximate syllogism' was a process that when given true propositions, yielded a new proposition that had at least a 95% probability of being correct."

    Why would we say that? We know what a syllogism is in its ideal form, and there's no "probability" about it.

    Let's take the syllogism in Barbara as an example: All men are mortal; Socrates is a man; therefore, Socrates is mortal. If the premises are true, the conclusion is true; the form of the argument is valid, full stop.

    Why do we have to bother imagining "approximations" to this form of reasoning that generate true conclusions from true premises only with 95% probability? What could that even mean?

    The only way I can think of offhand to "approximate" this form of reasoning is to have an undistributed middle term, in which case the argument is simply invalid even if the two middle terms are close in meaning: All men are mortal; Sherlock Holmes is a man; therefore Sherlock Holmes is mortal. Sherlock Holmes is a fictional man ("[w]ho never lived and so can never die," as Vincent Starrett famously wrote) and therefore not a "man" in the sense intended by the term in the first premise (which is why he can also be "immortal" in a sense different from that intended by that term in the first premise).

    And who cares? All Ross's (and Ed's) argument (have you even read it?) needs is that we know what a syllogism in Barbara is. The fact that we can fail is merely confirmation that there is such a thing as success.

    ReplyDelete
  131. @Crude,
    "While I'd like to see these "logically impossible interpretations" that are made logically impossible owing to the machine itself, I think your second paragraph contains a problem.

    You say that it must be possible for a physical system to exclude a non-empty class of interpretations, because otherwise we wouldn't be able to reason about "the machine itself". But based on what you're saying, it sounds like we're not reasoning about "the machine itself" - we're reasoning about interpretations related to the machine. If that's the case, then it doesn't seem as if it's necessary for a machine to exclude a non-empty class of interpretations - it's just necessary for one particular mental interpretation to exclude them. 'The machine itself' isn't the relevant party, so to speak."

    The problem is that I want the thing itself to be the relevant party. I want to be able to think about things with my abstractions, otherwise they are useless. I claim to be able to tell, at least in a large number of cases, from physical variables alone whether or not a thing is a squirrel, or whether my calculator is working as calculators do.

    These are no less abstractions than "triangle" or "modes ponens", except in so far in these cases it is very hard to leave (for instance) whether or not something is a squirrel up to totally arbitrary interpretation.

    If I Ross is offering me a choice between a perfect reason which cannot actually assert predicates about the real world and a reason which is not necessarily reliable, then I must say I prefer the later.

    Any physical object divides the world of interpretations into two classes, those which reasonably apply to it and those which do not. All I am doing is dualizing the process of judgement which we use when we apply our abstractions to actual objects. If one of the sets of interpretations is empty, if the physical system in question literally admits of any interpretation whatsoever, then the abstraction is a trivial one.

    To borrow Feser's example there may be some doubt as to what the delta symbol represents, but it does not represent the shape of a square.

    ReplyDelete
  132. @grodrigues,

    "What you are saying is that we would have programs which we do not, and cannot, know are correct but that would compute the correct answers; we would not know that computers work as they should. But if we were not, and could not, be able to know the programs were correct how could we ever be justified in believing that the program spitted out the correct answer?"

    I add to this only two caveats.

    (1) is it is possible to demonstrate that a program is correct without a completely cut and dry definition of the semantics underlying the programming language. All that is required is that the semantics obey certain invariants. The same of course is true if we back out the Curry-Howard isomorphism and make a similar statement about proofs. So abstraction is evidently not as definite as Ross wants us to believe.

    (2) The problem of "what does this program do" is stronger than the halting problem (since "it spins and spins forever" is one possible answer). So we cannot by inspection know what a program does. Yet, whether we know what it does or not, it does it! Which is to say we might expect there to exist cases in which the program actually does compute the correct answer (to some arbitrary question), yet we can't ever be certain that it does, even if we know the question very exactly. ( having written this I admit to being a little shocked by it. Can that possibly be right? check my math ).

    ReplyDelete
  133. @Scott,
    "I'm not at all sure what principles are being used to rule out possible "meanings" of physical states and behaviors, or even whether any such principles are possible."

    If I insist to you that a free electron is a very elegant translation of the last act of Hamlet, am I anything but totally mad?

    Between matter and reason there must be some bridge. Just because we can't find it doesn't mean it isn't there.

    ReplyDelete
  134. @Glenn and George
    I accept there is a difference between particular events and general likelihood but I consider it a difference in degree. There is a very strong connection between them. If someone is intoxicated we do not know which neural pathways are affected to the level of producing outputs drastically different from sobriety, we only know some of them are. Those particular changes are what correspond to various alterations in perception, memory or behavior and if that isn’t complicated enough, they are also going to be expressed relative to specific circumstances.

    If I insist to you that a free electron is a very elegant translation of the last act of Hamlet, am I anything but totally mad?

    Quite the contrary, you could be very profound. Isaiah 55:12 “For you shall go out in joy and be led forth in peace; the mountains and the hills before you shall break forth into singing, and all the trees of the field shall clap their hands."

    Also, a cool song.

    ReplyDelete
  135. Scott,

    "Huh? My point (and FZ's) was that we do and must know the ideal in order to tell whether we're approximating it."

    You *think* you do. But what if the concept 'syllogism' is itself incoherent or impossible to realise? If, as according to Ross, real syllogisms cannot be physically implemented, only something that coincidentally coincides in some of its properties, then it might still be perfectly possible to have an *idea* of a syllogism in exactly the same way as it is possible to have an *idea* of a conscious and intelligent machine that means what it says. You claim with Ross that the latter is impossible, but since we *know what it is* in order to be able to approximate it with AI, it must be real, right?

    Ross's argument seems to be assuming that counterfactuals aren't physically real. He might be right. The belief that there are things that could have happened but didn't is a product of our mental model of the world combined with our ignorance. We don't know what the precise initial conditions are, so we run the model on one set and get a certain outcome, then run them on a slightly different set and get a different outcome, and presume that either could have happened. But back in reality, there is only one actual initial state and one actual outcome, proceeding from it like clockwork. Potentiality is an illusion. Actuality is all there is.

    And since only one path was actually explored, *you can't tell* what would have happened on any of the other paths. So when you think you're applying modus ponens, you might have actually been applying a different rule that coincidentally coincided with modus ponens in that instance. Maybe you would have said Socrates was mortal whether the premises were true or not, since you happen to know that he's dead. Who can say? It didn't happen, so we'll never know.

    "Let's take the syllogism in Barbara as an example: All men are mortal; Socrates is a man; therefore, Socrates is mortal. If the premises are true, the conclusion is true; the form of the argument is valid, full stop."

    If the *form* of the argument is all that is required, it would be just as valid when machines did it. Ross says not.

    "And who cares? All Ross's (and Ed's) argument (have you even read it?) needs is that we know what a syllogism in Barbara is."

    Yes, I *have* read it. Or at least, Ross's paper and Ed's blog exposition. Ed's paper appears to be behind a paywall.

    And as I said, it's not clear to me why Ross is taking 'physical process' to mean just its history of states, rather than an implementation of certain rules of behaviour in conformity with physical law, nor why specifically he thinks the same argument doesn't apply to humans. For the former he doesn't say. For the latter he cites adverse consequences and incredulity.

    So far as I can see, both humans and machines can apply syllogisms the same way, by a rote rule-following process equivalent to a table look-up. It's true that the idea of following a rule of behaviour involves counterfactuals, since the rule defines what happens for all inputs, not just the ones you've got. So I can see that if Ross has a reason to exclude rules as non-physical, then it's true that what's left isn't enough to reason with. But I don't see why the same cannot be done to the humans too, which would leave them unable to reason as well.

    You seem to be arguing here that humans *do* know what rules and counterfactuals are. Well yes, I agree. But so do the machines. All the different alternatives and outcomes are *there*, listed in the look-up tables.

    ReplyDelete
  136. Is not equating the form of syllogisms and the form of the functions of machines just equivocation. The form of a syllogism is an abstract logical concept, whereas the form of machines is physical procedures that are meant to signify meaning.


    To make statements about machines knowing things seems to come very close to begging the question. After all, it seems to follow from Ross's arguments that machines cannot be said to know - or this is a natural implication one would draw.

    The adverse consequences you are referring to are essentially skepticism and the incoherence of all thought and argument, including those to support naturalism and to support the arguments against Ross's argument. I have noticed, in discussions of the argument from reason, that too many naturalists are dismayingly ready to throw away reason rather than naturalism, but I don't think Ross need show anything more than indeterminancy of formal thinking leads to incoherence and the break down of thought: my belief, and I understand this is a general belief, is that is asinine to entertain questions of skepticism in a discussion such as this.

    What defenders of Ross's argument could do is to copy C.S Lewis and Victor Reppert and make the point that the existence of logical inference is to be treated as self-evident. Then those wanting to veer off into total incoherence can be separated and do so elsewhere.

    ReplyDelete
  137. The problem with rule implementation is indeterminacy.

    Also, any meaning we attach to the physical does not change its physical properties.

    ReplyDelete
  138. "So far as I can see, both humans and machines can apply syllogisms the same way, by a rote rule-following process equivalent to a table look-up."

    Was the table made by someone who determinately grasped syllogisms?

    ReplyDelete
  139. @NiV:

    "If the *form* of the argument is all that is required, it would be just as valid when machines did it. Ross says not."

    Once again, that's not what Ross says, and your persistent misunderstanding of the point is becoming less and less defensible as this thread goes on.

    What Ross says is that a purely physical system doesn't in and of itself instantiate the abstract form of syllogistic reasoning at all. When a machine does instantiate that form (in an analogical way deriving from human intention), of course the form is valid.

    Did you honestly and seriously think Ross held that the formal validity of an argument depends on something more than its form?

    ReplyDelete
  140. (Instead of "the abstract form of syllogistic reasoning" I should have said "the abstract form of a syllogism in Barbara.")

    ReplyDelete
  141. Anonymous,

    "The physicalist need not accept such mutually exclusive categories, and this argument is not asking them to accept them. Or rather, these categories are only established as a consequence of the argument."

    I finally read through the paper and I see just the opposite. Ross made no attempt to justify those exclusive categories. He provides no evidence that one category doesn't absorb the other. If you see that evidence, please tell me where.

    ReplyDelete
  142. Jeremy,

    "The form of a syllogism is an abstract logical concept, whereas the form of machines is physical procedures that are meant to signify meaning."

    The behaviour of the machine, as required by the laws of physics, embodies the abstract logical concept.

    "To make statements about machines knowing things seems to come very close to begging the question. After all, it seems to follow from Ross's arguments that machines cannot be said to know - or this is a natural implication one would draw."

    To forbid making statements about 'machines knowing things' before settling the question would do the same.
    The question is, does it actually follow from Ross's argument? Is the argument valid?

    "I have noticed, in discussions of the argument from reason, that too many naturalists are dismayingly ready to throw away reason rather than naturalism,"

    As far as naturalists are concerned you have already done that by throwing away machine reasoning. But they try to be open-minded, and so will discuss for the sake of argument extremely unlikely hypotheses that they don't like or believe. It would be nice if everyone did the same. :-)

    "but I don't think Ross need show anything more than indeterminancy of formal thinking leads to incoherence and the break down of thought: my belief, and I understand this is a general belief, is that is asinine to entertain questions of skepticism in a discussion such as this."

    Ross's argument, when applied to human reasoning in the same way he applies it to machine reasoning, implies the incoherence and breakdown of thought. So either there is a *specific* reason why it does not apply to human thought, or the argument is invalid, or reason breaks down. You and I agree that reason breaking down would be very bad and is almost certainly not true, but you still have to go through the formal process to show exactly how and why it doesn't. And that means you have to entertain the possibility hypothetically until you have shown how it can be rejected.

    What I'm after is the *specific* reason how and why the argument does not apply to humans. Because I'm thinking that if the argument *does* apply equally well to humans, then there almost certainly must be something wrong with it, since I quite agree that humans *can* reason.

    Anonymous,

    "The problem with rule implementation is indeterminacy."

    Do you mean indeterminacy in Ross's sense of many functions being consistent with a given history of outputs? Or Feser's sense of symbols having more than one possible meaning?

    "Also, any meaning we attach to the physical does not change its physical properties. "

    Quite so. So the meaning must be inherent in the physical properties. The question is, is the *transition rule* that a physical system follows one of its physical properties?

    "Was the table made by someone who determinately grasped syllogisms?"

    No, it was generated automatically. :-)

    Scott,

    How is
    "What Ross says is that a purely physical system doesn't in and of itself instantiate the abstract form of syllogistic reasoning at all"
    consistent with
    "When a machine does instantiate that form [...] of course the form is valid."
    ?

    Either it can instantiate it it or it can't.

    "Did you honestly and seriously think Ross held that the formal validity of an argument depends on something more than its form?"

    Yes. That's what this whole argument is about, and why I disagree. Any computer can print out syllogisms by the score, all of which conform to the *form* of the syllogism. Ross requires something non-physical in addition. Or at least, so I read him.

    ReplyDelete
  143. "So the meaning must be inherent in the physical properties."

    How does this follow?

    "No, it was generated automatically. :-)"

    By chance?

    ReplyDelete
  144. I think you should have linked to this, Jeremy.

    http://edwardfeser.blogspot.com/2012/02/popper-contra-computationalism.html

    The relevant part is under:

    "There are significant differences between these writers’ respective statements of the argument, but a “generic” version might go as follows:"

    ReplyDelete
  145. NiV,

    How is
    "What Ross says is that a purely physical system doesn't in and of itself instantiate the abstract form of syllogistic reasoning at all"
    consistent with
    "When a machine does instantiate that form [...] of course the form is valid."
    ?


    Were he alive today, Charles Babbage might chime in as follows: "[Though] I am not able rightly to apprehend the kind of confusion of ideas that could provoke such a question[, I do notice an eerie similarity to an earlier confusion of ideas regarding a prior pair of statements]." (See 2nd para of p 67 here.)

    Either it can instantiate it it or it can't.

    A purely physical system can instantiate the abstract form of syllogistic reasoning -- in an analogical way deriving from human intention.

    A purely physical system cannot instantiate the abstract form of syllogistic reasoning -- in and of itself.

    To put it another way: though in and of itself it cannot, in an analogical way deriving from human intention it can.

    But Scott has already said as much.

    Potentiality is an illusion. Actuality is all there is.

    This seems to drive a stake through the heart of whatever hope some may have entertained regarding the future state of your understanding.

    ReplyDelete
  146. Professor,

    My latest response (an apology) is up at my blog.

    ReplyDelete
  147. @NiV:

    "Yes. That's what this whole argument is about, and why I disagree. Any computer can print out syllogisms by the score, all of which conform to the *form* of the syllogism. Ross requires something non-physical in addition. Or at least, so I read him."

    [sigh] I give up. It doesn't seem to matter how many different times and ways we try to summarize the point for you, you just aren't going to get it.

    But no, that is not what this argument is about, and no, you're not only not reading Ross correctly, you're persistently reading your own nonsense into him.

    @Glenn:

    "This seems to drive a stake through the heart of whatever hope some may have entertained regarding the future state of your understanding."

    Indeed.

    ReplyDelete
  148. @Robert Oerter:

    Kudos for your gracious blog post.

    ReplyDelete
  149. Step2,

    @Glenn and George
    I accept there is a difference between particular events and general likelihood but I consider it a difference in degree. There is a very strong connection between them. If someone is intoxicated we do not know which neural pathways are affected to the level of producing outputs drastically different from sobriety, we only know some of them are. Those particular changes are what correspond to various alterations in perception, memory or behavior and if that isn’t complicated enough, they are also going to be expressed relative to specific circumstances.


    Were I to aver that "8 + 8 = 10" is provably true, you wouldn't need to know anything about neural pathways in order to determine whether my averment is rational.

    All you would need to know is my answer to your question, "In what base?"

    And if my answer is, "16,", then, as a rational person yourself, you would be duty-bound to acknowledge that, regardless of how tight, looped or ossified I might be, my averment is indeed rational.

    This isn't to say that the connection you call attention to doesn't exist, only that it isn't relevant to the point that George was making.

    ReplyDelete
  150. "But Ross is doing no such thing. He would be begging the question in a way parallel to Oerter’s blatant begging of the question only if the further premise he needed was the premise that Hilda is not purely physical. But that is not the premise he appeals to, and it is not the premise he needs. Rather, what he needs and what he appeals to is the further premise that Hilda engages in formal thinking. That premise together with A, B, and C is what generates the conclusion -- not a question-begging assumption but rather a demonstrated result -- that Hilda is not purely physical."

    Did people read this part?

    ReplyDelete
  151. Anonymous,

    "How does this follow?"

    Sorry, I should have explained. I'm talking about the meaning in a purely physical system, such as a computer. Since there is nothing there but physical properties, the meaning must be inherent in the physical properties. But we already went through that one.

    "By chance?"

    No.

    Glenn,

    "A purely physical system can instantiate the abstract form of syllogistic reasoning -- in an analogical way deriving from human intention."

    Either it instantiates it or it doesn't. You seem to be agreeing that it can instantiate it. But the next moment you say it can't. Does it or doesn't it?

    "A purely physical system cannot instantiate the abstract form of syllogistic reasoning -- in and of itself."

    Why not?

    "his seems to drive a stake through the heart of whatever hope some may have entertained regarding the future state of your understanding."

    I've asked several times why Ross takes this approach - of ignoring the way a machine mechanically instantiates a transition rule and instead treats it as a history of states/outputs - and got no answer. If nobody will explain, I'll naturally have to draw my own conclusions.

    "And if my answer is, "16,", then..."

    ...we would have to ask you "And what base is *that* in?"

    Scott,

    "[sigh] I give up. It doesn't seem to matter how many different times and ways we try to summarize the point for you, you just aren't going to get it."

    Perhaps that's the problem. You keep on trying to summarize the *point* or conclusion, but what I need a summary of is the *argument*. You keep on making complex statements as if they're obvious, but which make no sense to me. And statements that seem obvious to me evidently make no sense to you.

    It's going to need a lot more time.

    ReplyDelete
  152. "Since there is nothing there but physical properties, the meaning must be inherent in the physical properties."

    1) There is nothing but physical properties in a computer.

    2) ???

    C) Meaning must be inherent in the physical properties.

    Help me out here.

    ReplyDelete
  153. From the original paper:

    "A decisive reason why a physical process cannot be determinate among incompossible abstract functions is "amplified grueness": a physical process, however short or long, however few or many outputs, is compatible with counterfactually opposed predicates; even the entire cosmos is. Since such predicates can name functions from "input to output" for every change, any physical process is indeterminate among opposed functions. This is like the projection of a curve from a finite sample of points: any choice has an incompatible competitor.

    We have no doubt that the processes in a mechanical adding machine and in a personal computer are entirely physical. Addition cannot be identical with either of those physical processes because then it could not be done by the other. Suppose that addition is identical with a function among those processes. Then the processes would have to determine that function to the exclusion of every incompossible function. But they cannot do that, as the "quus," "grue," and "points-on-a-curve" examples show. So the machines cannot really add."

    ReplyDelete
  154. @NiV:

    "[W]hat I need a summary of is the *argument*."

    In other words (my words), what you need is a summary of what the argument is supposed to show and how it's supposed to show it: i.e., as I said, its point. We've given you that quite a number of times, but okay, I'll try once (and only once) more.

    (1) We know that the intellect is able to instantiate pure functions and forms. We know that because, for example, we know that when we add a column of figures, we're trying to carry out the mathematical operation of addition—and even if we fail, we still know what that operation is (as we must in order to know we're failing to perform it correctly) and our intellect therefore still must instantiate it.

    (2) But a strictly physical system does not in and of itself "instantiate" (say) the form of a syllogistic argument, because it is simultaneously consistent with multiple incompossible such forms and therefore determinately instantiates none of them.

    Note that at this point in Ross's argument, he's quite specifically denying that "[a]ny computer can print out syllogisms by the score, all of which conform to the *form* of the syllogism." No, it can't. All it can do is spit out markings or images that we human beings may, if we wish, interpret as "conform[ing] to" that form.

    This brings us to a side point. There seems to be another sense in which a machine might be said to conform to such a form: in a certain analogical or derivative sense, an adding machine (for example) can be said to "add." However, even if this is true, it's irrelevant to Ross's argument because it's entirely cashed out in terms of human intentions rather than in terms of the physics of the machine. And it doesn't matter if it's false; all Ross argues is that the machine can instantiate a pure form in at most this analogical or derivative way. If it doesn't even do that, then so much the better for his argument.

    (3) Therefore, intellect can't be purely physical. Q.E.D.

    I don't think you'll see a single statement in that summary that you haven't seen from several of us before in some form or another. That's why it puzzles me to find you still expressing fundamental misunderstandings like Ross thinks something non-physical has to be added to the form of a syllogistic argument in order for it to be valid.

    ReplyDelete
  155. Anonymous,

    "Help me out here."

    A) There is (hypothetically) meaning in a purely physical computer.
    B) There is nothing but physical properties in a computer.
    C) The meaning must be inherent in the physical properties.

    "From the original paper:"

    Thanks! This is much more helpful!

    "a physical process, however short or long, however few or many outputs, is compatible with counterfactually opposed predicates"

    What does it mean by describing a physical process as "short or long"?

    If we define a physical process as an initial state and a transition rule, is that a short physical process or a long one?

    "Addition cannot be identical with either of those physical processes because then it could not be done by the other."

    Why?

    What's the distinction between 'is the process' and 'done by the process'?

    "Suppose that addition is identical with a function among those processes"

    We can add pebbles to a pile of pebbles. We can add apples to a pile of apples. If we add 3 pebbles to 2 pebbles we get 5 pebbles. If we add 3 apples to 2 apples we get 5 apples. Same if we add 3 eggs to 2 eggs, or 3 goats to 2 goats, or whatever. We cannot add 3 apples to 2 apples and get 7 apples - physics doesn't work that way.

    Each is an instantiation of addition; we can use *any* of them to calculate the answer to *all* of them.

    "But they cannot do that, as the "quus," "grue," and "points-on-a-curve" examples show."

    How do the quus and grue examples apply to adding 3 apples to 2 apples? Is Ross saying that if you physically add 68 apples to 57 apples you could *actually get* 5 apples? Is that what Ross means?

    If not, what *does* he mean?

    ReplyDelete
  156. (It's also why some of us are puzzled by your repeated insistence that "[e]ither [a physical system] instantiates [a pure form or function] or it doesn't. You seem to be agreeing that it can instantiate it. But the next moment you say it can't. Does it or doesn't it?" Surely this is a point that should be clear to you by now. The answer is: no, it doesn't, except in at most a derivative or analogical sense that is irrelevant to the main argument.)

    ReplyDelete
  157. @NiV:

    "If not, what *does* he mean?"

    I think it's clear at this point that you need a good deal more than a "summary of the argument."

    ReplyDelete
  158. Scott,

    Thank you.

    "Surely this is a point that should be clear to you by now. The answer is: no, it doesn't, except in at most a derivative or analogical sense that is irrelevant to the main argument.)"

    Good. Now what do you mean by "a derivative or analogical sense"?

    I add 3 beans to 2 beans and get 5 beans, and suggest that this 'means' that 3 of anything added to 2 of the same thing gives 5 of those things. What is this 'derivative or analogical' of?

    Would it be untrue if there were no people?

    ReplyDelete
  159. A bead on an abacus can represent 1, an identical bead can represent 10.

    With regards to the apples, here's one way to think about it. Suppose you are trying to do -57 + 68. One way of doing it is gathering 57 apples on one side, and 68 apples on the other. The 57 apples each represent -1's, and the 68 apples on the other side represent +1's. Begin by moving one apple from each side to the middle. So far you have -1 and +1, which is zero. Continue that action until you have 57 apples from one side and 57 apples from the other side in one big pile. You are still at 0, because -57 + 57 is 0. Now, continue moving apples to the center. This time, it is +1, +1, +1, etc. Once all the apples are moved to the center, you will be at +11.

    ReplyDelete
  160. @NiV:

    "I add 3 beans to 2 beans and get 5 beans, and suggest that this 'means' that 3 of anything added to 2 of the same thing gives 5 of those things. What is this 'derivative or analogical' of?"

    Your unwillingness and/or inability to engage Ross's argument on its own terms even while claiming to be trying to understand it.

    ReplyDelete
  161. FZ,

    Yes, indeed. That's just the sort of thing I'm thinking of.

    Scott,

    "Your unwillingness and/or inability to engage Ross's argument on its own terms even while claiming to be trying to understand it."

    Yes, so you've said, repeatedly.

    So what precisely does 'derivative or analogical' mean here? When a device adds by adding one pile to another, what is this 'derivative or analogical' of?

    ReplyDelete
  162. "Is Ross saying that if you physically add 68 apples to 57 apples you could *actually get* 5 apples? Is that what Ross means?"

    I'm still working through the paper myself, but I think Ross would say that your scenario of 68 apples joining with 57 apples would also satisfy the abstract function "wadding" (I made that name up for the sake of discussion) where "wadding" is described as:

    x wad y = x + y, if x, y < 200, =5 otherwise

    ReplyDelete
  163. In other words, in your scenario, we expect 125 apples, and we actually do get 125 apples. But Ross would say that the scenario is compatible with both adding and wadding. He would ask "How do we decide which abstract function was instantiated?"

    ReplyDelete
  164. "but I think Ross would say that your scenario of 68 apples joining with 57 apples would also satisfy the abstract function "wadding""

    Yes, that's what I suspect he would say, too.

    Except that I would say that the physical process of adding 68 to 57 apples has two aspects to it: the specific numbers of apples involved, and the device that pushes one heap into the other. Is the device doing 'wadding'? Can it do anything other than 'adding'?

    This is what I mean when I say Ross appears to be treating the sequence of states and outcomes as 'the physical process', whereas I would be looking at the device for pushing one heap onto another. I don't see how the *device* can be be doing anything other than adding.

    Do you see what I mean?

    ReplyDelete
  165. We seem to have returned to the addition is a physical process nonsense. Wasn't this dealt with already?

    This isn't any sort of intellectually serious attempt to understand what Ross's own argument means. To whatever extent it's more than mere confusion, it seems to be an attempt to make a counterargument under cover of passively pretending not to understand. Wait, I'm confused. Ross couldn't possibly mean that, because I don't agree with it. So does he mean this other thing instead? I just don't get it!

    Ross does not regard there being six rocks over here and there being five rocks over there, together with their entailment of there being eleven rocks in total, as an example of "the pure function of addition." Nor would he regard it as becoming "the pure function of addition" just because a machine shoved the two piles of rocks together.

    Even you, NiV, regard it as "addition" only by equivocation. What you really seem to have in mind is something more like taking the union of two non-intersecting sets (and perhaps counting the elements in that union?)—and even that, as grodrigues pointed out some time ago, isn't a physical process either, never mind that it isn't "addition" in the requisite sense.

    For present purposes it's sufficient to say that "addition" is a mathematical operation that takes two numbers and returns their sum. You may say, if you like, that combining two piles of rocks is in some way "adding" one to the other, but it's very obviously not what Ross means and you will not advance your (or anyone else's) understanding of Ross's argument by thinking of things in those terms.

    But then, I think you know that already.

    ReplyDelete
  166. donjindra,

    How about you point out how the argument requires what you claim, and leave your dubious exegesis of Ross for another time.

    ReplyDelete
  167. Scott,

    "... it seems to be an attempt to make a counterargument under cover of passively pretending not to understand."

    I'm not pretending, but yes, I am making a counter-argument. The aim of which is to determine what the argument means by asking where the counter-argument goes wrong.

    "Ross does not regard there being six rocks over here and there being five rocks over there, together with their entailment of there being eleven rocks in total, as an example of "the pure function of addition." Nor would he regard it as becoming "the pure function of addition" just because a machine shoved the two piles of rocks together."

    OK, good. That's definite enough. Now I just have to figure out why.

    "Even you, NiV, regard it as "addition" only by equivocation."

    Not at all. I regard that sort of addition as the physical phenomenon that mathematical addition seeks to reproduce. And I regard neurons shuffling signals as just another physical process implementing the same function. But you already know that, and this isn't about what I think.

    "For present purposes it's sufficient to say that "addition" is a mathematical operation that takes two numbers and returns their sum."

    Good. Apart from the word "mathematical", which might or might not mean anything, that's exactly what my device for pushing two piles together does.

    "You may say, if you like, that combining two piles of rocks is in some way "adding" one to the other,..."

    Great!

    This is just what I was looking for!

    "... but it's very obviously not what Ross means and you will not advance your (or anyone else's) understanding of Ross's argument by thinking of things in those terms."

    Good.

    So the reason for the difference of opinion is that we have different definitions of 'addition'. Machines are capable of my sort of 'addition', but are not capable of Ross's sort of 'addition'. (I presume, since I'm still not quite sure what Ross's definition is. But I'm not going to worry about it.)

    So I don't see it as a problem for the materialists. All they have to do is use the other definition for addition, which I suspect they would have done anyway, and the problem goes away. Everybody's happy!

    Thanks for clearing that up. Most helpful. :-)

    ReplyDelete
  168. @NiV:

    "So I don't see it as a problem for the materialists. All they have to do is use the other definition for addition, which I suspect they would have done anyway, and the problem goes away."

    That would be a nice trick if you could pull it off. But the argument that physical processes don't instantiate pure functions isn't conjured into oblivion by a decision about word usage. Not using this or that word to refer to them doesn't make them go away.

    ReplyDelete
  169. NiV,

    On naturalists and reason, I don't think it shows openmindedness that quite a few naturalists are willing to throw away reason before naturalism. Naturalism is not so well supported that, when confronted with evidence it undermines reason, one would sensibly prefer naturalism. Indeed, because it is a philosophical position, and therefore to be supported by reason itself, it would have to be essentially self-evident before it would make sense to prefer naturalism to reason. That not a few naturalists try this trick, aside from being asinine and a sort of sophistical argumentative trick in many instances, is a sign not of an openmind, but that their belief in naturalism is in fact dogmatic and not the reasoned and evidence based perspective they claim.

    ReplyDelete
  170. "Is the device doing 'wadding'? Can it do anything other than 'adding'?"

    I'm not seeing your point here, can you elaborate? The machine's pushing is compatible with adding and wadding:

    The machine pushes (adds) 57 to 68 to get 125.

    The machine pushes (wadds) 57 to 68 to get 125.

    Where does the differentiation come from?

    ReplyDelete
  171. @FZ:

    "I'm not seeing your point here, can you elaborate?"

    I think what NiV has in mind is that by looking at the design of the machine, s/he can tell that it can only ever perform a physical operation that we can describe as "adding."

    Unfortunately for NiV, that's not true. Any such machine will have an upper limit Nmax on the number A of rocks (of any given minimum weight or size that we decide to count as "rocks") it can push at one time, and so its physical operation will be compatible with any version of "wadding" of the form

    A ⊕ B = / A + B for A <= Nmax
                  \ something else otherwise (for example, Nmax + B)

    and it won't be commutative (since the upper limit applies only to the pile being pushed, not to the pile being pushed into).

    ReplyDelete
  172. (Or at the very least there will be some number of rocks too large for it to push.)

    ReplyDelete
  173. @NiV:

    "Ross's argument, when applied to human reasoning in the same way he applies it to machine reasoning, implies the incoherence and breakdown of thought. So either there is a *specific* reason why it does not apply to human thought, or the argument is invalid, or reason breaks down. . . .

    What I'm after is the *specific* reason how and why the argument does not apply to humans."

    And how many times do you insist on having that reason given to you? Robert Oerter himself has graciously come here and apologized for overlooking this very point. Ross provides a non-question-begging argument that humans are not purely physical. This has been stated time and again; shall we send it to you engraved on a plate of gold?

    ReplyDelete
  174. Scott,

    "That would be a nice trick if you could pull it off. But the argument that physical processes don't instantiate pure functions isn't conjured into oblivion by a decision about word usage."

    No, of course not. But somebody using the other definition doesn't have to worry about it. Somebody proving that functioning computers don't exist would be worrying. Somebody proving that functioning computers defined in a peculiar abstract way that none of them agree with don't exist is perfectly fine. We'll carry on using the computers we've got, that work perfectly well in our sense, and you can rest satisfied that you've proved they don't exist or don't work, in your sense. Both statements and situations are true, in parallel.

    Jeremy,

    "On naturalists and reason, I don't think it shows openmindedness that quite a few naturalists are willing to throw away reason before naturalism."

    Naturalists are willing to throw away *both*, hypothetically, to see where the argument goes.

    "Naturalism is not so well supported that, when confronted with evidence it undermines reason, one would sensibly prefer naturalism."

    That's a matter of opinion.

    But in formal reasoning it doesn't *matter* what anyone prefers, it matters what you can and can't prove. Can you *prove* that reason is not undermined by the argument? No? Then you've got a problem.

    The complaint is that naturalists let their desires overrule their reason, and reject valid arguments simply because they would challenge naturalism. So, OK, we can all agree to put our opinions to one side, and see where the logic leads. But you can't then suddenly reintroduce them when it goes somewhere *you* don't like. You have to play the game fairly.

    Anyway, we have strong evidence that human reasoning actually is quite flaky. Paradoxes, fallacies, the difficulty most people have with mathematics at school, public discourse, the Wason selection task, ... It's good, and a lot better than the cynics would say, but it's a long way from perfect. And we know that for a fact.


    FZ,

    "I'm not seeing your point here, can you elaborate? The machine's pushing is compatible with adding and wadding:"

    The machine pushes the contents of one bucket into the other bucket. It does so, however many objects are in the bucket.

    The mechanics of the way the device is arranged define the result you'll get for *any* initial state. It defines it for all of them simultaneously.

    If your function 'wadding', 'xadding', 'yadding' etc. differs at any point from 'adding', which function would the machine do at that point? You don't have to actually try it out to know. The mechanism itself and the way it works will tell you.

    Scott,

    Yes, that expresses it very well.

    "Unfortunately for NiV, that's not true. Any such machine will have an upper limit Nmax on the number A of rocks (of any given minimum weight or size"

    Who says we're doing this at the bottom of a gravity well? :-)

    Anyway, I expect if you asked a human to add two randomly selected integers with 10^(10^100) digits, they'd have a bit of difficulty, too... Brains have a finite number of states as well.

    But it's a good point.

    It depends whether you count "I don't know" as a correct answer, I guess.

    "And how many times do you insist on having that reason given to you? Robert Oerter himself has graciously come here and apologized for overlooking this very point."

    Yes I read it. He didn't say he *agreed* with Feser's argument, he only said (in effect) that whatever faults it might have, *question begging* wasn't one of them. He says: "I hope to return to my original epistemological objection (as time permits)".

    Although I do like that 'himself'! Why would it matter to me what Robert Oerter thinks?

    ReplyDelete
  175. Thanks for the example Scott. I guess I still need to look into Ross and Kripke.

    One last question NiV, what if the total mass of the rocks to be pushed is equivalent to, say, the Earth? Wouldnt that create a gravity well?

    ReplyDelete
  176. FZ,

    "One last question NiV, what if the total mass of the rocks to be pushed is equivalent to, say, the Earth? Wouldnt that create a gravity well?"

    Yes. Indeed, that would be one way to perform the addition. How would you add the mass of this giant asteroid to the mass of that unsuspecting planet...? ;-)

    Not all mechanisms need be man made.

    Scott has a valid point that for any given machine, there are always numbers too big for it to add. So 'pure addition' - the sort that can add numbers of 10^10^10^10^...^10 digits in length - cannot be physically implemented by any real mechanism. My reply would be that exactly the same sort of limit applies to humans, too. In fact, I'd say that machines are rather better at adding big numbers than humans are. I don't think it gets us anywhere.

    I don't feel that the objection gets to the heart of the issue. It's a problem with the specific example chosen, but it would be easy to pick different examples of functions on finite domains where it wasn't.

    ReplyDelete
  177. Anonymous.

    "How about you point out how the argument requires what you claim, and leave your dubious exegesis of Ross for another time."

    I think it's obvious Ross needs to justify the arbitrary nature of his language. But for elaboration I propose the following.

    This is Feser's condensed form of the proof:

    All formal thinking is determinate
    No physical process is determinate
    Thus, no formal thinking is a physical process.

    My objection is that formal thinking and physical process are assumed to be mutually exclusive throughout Ross's paper and he never considers the possibility that formal thinking is a subset of physical process.

    I claim the condensed form of the proof is actually closer to this:

    All formal thinking is determinate
    No lifeless physical process is determinate
    Thus, no formal thinking is a lifeless physical process.

    That conclusion is not as controversial and is closer to reality. The main objection would come from those in the artificial intelligence community. So they might suggest:

    All formal thinking is determinate
    No non-intelligent physical process is determinate
    Thus, no formal thinking is a non-intelligent physical process.

    IOW, Ross could exchange non-intelligent physical process or lifeless physical process for physical process throughout his paper with no other changes and it wouldn't affect his line of argument at all. This tells me he has simply masked out possibly relevant properties from his categories by choice of language. And that languagr makes his conclusion seem a lot more than it really is.

    ReplyDelete
  178. I've just started reading ”Aquinas” by Edward Feser. I have problem understanding form matter relation.
    Consider the quoted example of red rubber ball. If the red rubber ball is melted it is said the change is essential as it becomes a puddle of goo, which has its own distinct form. However when the same red rubber ball is painted over with a blue paint the change is accidental because the ball form hasn’t been affected.
    But the original form is not of a rubber ball, but RED-RUBBER-BALL. If we paint that red rubber ball blue then we made a new object whose form is BLUE-RUBBER-BALL and considering it as less essentially different from the original than the puddle of goo is, to my mind, arbitrary. So why changing the shape of the matter from ball to puddle (or gas) is essential while changing the colour is from red to blue (or yellow) is accidental? Can someone, please, explain? Thanks.
    Thomas H.

    ReplyDelete
  179. donjindra,

    "My objection is that formal thinking and physical process are assumed to be mutually exclusive throughout Ross's paper and he never considers the possibility that formal thinking is a subset of physical process."

    A subset of a physical process is still a physical process. Thus, the premise still applies to it.

    ReplyDelete
  180. @NiV:

    "Why would it matter to me what Robert Oerter thinks?"

    Because you wrote, "What I'm after is the *specific* reason how and why the argument does not apply to humans," and because Oerter had pretty much the same problem and now admits having overlooked the answer in his own "Hilda" counterexample, which he has now discarded.

    In other words, because it's relevant to your question. If you don't care about that, I guess I don't either.

    ReplyDelete
  181. "So why changing the shape of the matter from ball to puddle (or gas) is essential while changing the colour is from red to blue (or yellow) is accidental?"

    Basically because we're regarding the object as a ball, because that's what it was designed to be. A ball is always (approximately) spherical, but it can be any color.

    This example (which is intended primarily as an illustration of the concept) might make it appear that what something is is just a matter of definition, but that's not generally the case. Balls in the sense relevant to this example are artifacts, man-made objects, which have the purposes we give them and which therefore might appear a bit arbitrary.

    But try the same thing with another of Ed's favorite examples: a squirrel. Paint it blue and you still have a squirrel. Melt it and you don't. (Not that being a squirrel is solely or even primarily a matter of shape, but you get the idea.)

    ReplyDelete
  182. @FZ:

    "My reply would be that exactly the same sort of limit applies to humans, too."

    And you would be right. The difference, again, is that we know we're trying to add, and in order even to know that we're failing (as e.g. "Those numbers are just too big for me to add"), we still have to know what addition is. Thus (and this is the real point) we know that we—our intellects, that is—are capable of formal thinking.

    And again, we needn't get too hung up on whether machines are "really" capable of instantiating pure functions or formal operations. All Ross's argument requires is that if any apparently physical system does instantiate a pure function, that isn't ever just a matter of the physics.

    For example, a Platonist might well take NiV's rock pile builder as instantiating (or even "participating in") the abstract mathematical principles of addition. (I think Jeremy Taylor made a remark along those lines.) But that wouldn't be a matter purely of physics either and it would still support Ross's argument.

    Also, even an Aristotelian-Thomist might be skeptical that in this or that instance a particular machine "really" instantiated the pure function of addition even in a derivative/analogical sense. Someone else might even think that sense was ridiculous anyway. That doesn't matter to Ross's argument either, which really says only that a purely physical system can instantiate a pure function at most in such a derivative or analogical way. As I've said elsewhere, if it can't even do that, then so much the better for Ross's argument.

    ReplyDelete
  183. Thank you, Scott.

    I understand your point of ball being an artefact, or designed to have a function. It is we who decide what the thing is whether or not we care to give it a definition. I think you are right.

    But let's consider possibility of red balls occurring spontaneously in a given physical environment. We could look at such an object both as a ball with property of redness, or redness with property of "ballness". Would painting over and melting such an object be different as far as the distinction of essential versus accidental is concerned?
    I hope I express myself clearly. English is not my first language.

    Thomas H.

    ReplyDelete
  184. Scott, I think that was NiV's comment.

    But I think I get the idea. I guess this is related to Ross' talk of "long or short"?

    ReplyDelete
  185. Scott, please excuse me bugging you again, but there is another thing about form-matter which is not clear to me.

    E.Feser says about Aristotelian doctrine of hylemorphism that it in essence means that "the ordinary object of our experience are composites of form and matter."

    I have problem with the word "composites". As I can not conceive of one existing without the other then what was the reason and purpose for Aristotle to introduce these concepts?
    But if form, being a "composite" exist separately from matter then has the form of, for example, red rubber ball, existed before man has actually produced the first red-rubber-ball?

    Thomas H.

    ReplyDelete
  186. @Anon:

    "But let's consider possibility of red balls occurring spontaneously in a given physical environment. We could look at such an object both as a ball with property of redness, or redness with property of 'ballness'. Would painting over and melting such an object be different as far as the distinction of essential versus accidental is concerned?"

    If that were all the information we had, I'm not sure we could tell. For example, I think we'd probably say that being composed of gneiss was more "essential" to a certain rock than having such-and-such a shape. On the other hand, we'd probably say that being round was more "essential" to a planet than being composed of such-and-such materials. (Planets don't have to be made of anything in particular, but being round, though not a fundamental property, is pretty closely related to the gravitation conditions that make it a planet in the first place.) The case of the naturally-occurring red balls might be analogous to either of these, depending on just what they were, how they came to be, and how they behaved.

    @FZ:

    "Scott, I think that was NiV's comment."

    Oops, so it was. I saw the "FZ" at the top of the part of the post addressed to you and mistook it for the poster's screen identity.

    "I guess this is related to Ross' talk of 'long or short'?"

    I think so. Here's what Ross says: "[A] physical process, however short or long, however few or many outputs, is compatible with counterfactually opposed predicates; even the entire cosmos is. Since such predicates can name functions from 'input to output' for every change, any physical process is indeterminate among opposed functions. This is like the projection of a curve from a finite sample of points: any choice has an incompatible competitor." He's saying that a purely physical process, no matter how long it goes on, is never sufficient in and of itself to determine just one pure function or operation.

    ReplyDelete
  187. I see that the moniker "qavistas" I made for myself long long-ago is still active.
    Will sign with it in the future.

    Thomas H.

    ReplyDelete
  188. @qavistas:

    "E.Feser says about Aristotelian doctrine of hylemorphism that it in essence means that 'the ordinary object of our experience are composites of form and matter.'

    I have problem with the word 'composites'. As I can not conceive of one existing without the other then what was the reason and purpose for Aristotle to introduce these concepts?"

    Aristotle agrees that neither exists without the other. But we can still distinguish between them conceptually for the purposes of understanding, just as we can distinguish between the north and south poles of a magnet even though neither can exist on its own.

    Why make the distinction? To put it intuitively, being a red ball isn't the same as being this red ball. For the latter we need something else, something that "individuates" this particular ball. Aristotle calls this "matter."

    "But if form, being a 'composite' exist separately from matter then has the form of, for example, red rubber ball, existed before man has actually produced the first red-rubber-ball?"

    This is actually a pretty contentious issue and it involves some important differences between Aristotle and Plato. To make a long story short, Aristotelian Thomism holds that although forms don't exist on their own, they do "pre-exist" in a certain analogical manner in the Divine Intellect. If there are (or even could be) red rubber balls, then God must have been "thinking" about red rubber balls from eternity.

    This post of Ed's might help, and so might at least parts of this one.

    ReplyDelete
  189. (I should also add that according to A-T, some forms do exist, or are at least capable of existing, without matter—angels and human souls being the most obvious two examples.)

    ReplyDelete
  190. Thank you so much, Scott.
    Let's see if I can and, if I do, how long will it take me to digest and understand the material the links you sent refer me to.

    BTW., I hope Thomas proves existence of God without referring to hylemorphism.

    "...according to A-T, some forms do exist, or are at least capable of existing, without matter—angels and human souls being the most obvious two examples"

    If they exist at all that is.

    ReplyDelete
  191. donjindra,

    You're argument is nonsense. The whole point of Ross's argument is to discover whether formal thinking can be a physical process. He simply does not assume they are mutually exclusive or he wouldn't have taken the time to write his paper. You haven't even begun to show that it is all an elaborate rouse and he is really engaged in a massive bout of question begging.

    The point of and reason for your insertion of qualifiers behind the term physical process is hard to see.

    ReplyDelete
  192. NiV,

    No, I'm sorry, you're reply about skepticism and naturalism is nonsense. Naturalism is a complex, philosophical position supported by rational speculation. It nothing like self-evident, or even so overwhelmingly evidenced that one should waste time speculating on preferring it to reason. After all, it is supported by reason and therefore preferring it to reason means not being able to rationally support naturalism.

    It is therefore simply asinine to to do anything but treat skeptical implications as undermining naturalism in normal arguments about the validity of naturalism. If someone is arguing for something and one points out that their argument being true would end in total incoherence, it is quite acceptable, even necessary, to point out their argument has failed. Now, naturalists are free to continue their speculation about naturalism undermining reason amongst themselves if they wish to, but it has no place in a proper argument critiquing naturalism and you have said nothing to suggest it does; you have simply not addressed the basic incoherence involved or why one would accept such incoherence for such a speculative, metaphysics as naturalism?

    That some naturalists will entertain its skeptical possibilities in settings like this is evidence not of openmindedness but of dogmaticism and sophistry.

    ReplyDelete
  193. @Jeremy Taylor and NiV:

    "Naturalism . . . is supported by reason and therefore preferring it to reason means not being able to rationally support naturalism."

    I must express my concurrence with this and even add that the point extends to any position whatsoever for which rational support is urged. Can you *prove* that reason is not undermined by the argument? is just about the most self-undermining question there could possibly be, if there is such a thing at all.

    ReplyDelete
  194. Jeremy,

    It's nothing to do with the desirability, complexity, whatever of naturalism. The same would apply to any conclusion. If you have an argument that apparently leads to incoherence, you have to know where the argument goes wrong, or you'll be left with a flaw in your understanding. It's not enough to say "I know the argument must be wrong, I'm not even going to consider it any more."

    I'll give another example. Some mathematicians have a childlike delight in apparent proofs of crazy things, like that 1 = 0. They present a sequence of apparently 'textbook' steps from obvious premises to the crazy conclusion. There's an error along the way, but you missed it.

    Now, anyone watching *knows* there's an error. 1 is not equal to 0. But they also know they missed it, which implies there is a flaw in their understanding. There's some basic mathematical operation they think is valid but which isn't. So what if they've used it in other proofs where the falsity of the conclusion *wasn't* given away by being so obviously wrong? It's vital for their peace of mind to understand what went wrong, and then to re-check their other conclusions to make sure the fatal step wasn't used.

    Simple example:
    Calculator says:
    1 / -1 = -1 / 1
    Apply same operation to both sides:
    Sqrt(1 / -1) = Sqrt(-1 / 1)
    Square root distributes over division:
    Sqrt(1) / Sqrt(-1) = Sqrt(-1) / Sqrt(1)
    Multiply both sides by Sqrt(1) Sqrt(-1) to clear fractions
    Sqrt(1) Sqrt(1) = Sqrt(-1) Sqrt(-1)
    Use the definition of a square root:
    1 = -1

    Did you spot the invalid step?

    Now, it's also possible to construct such a flawed argument so that instead of proving something incoherent, it proves that either an arbitrary conclusion is true *or* something incoherent. This gives you a most dangerous get out.

    For example:
    Let S be the sentence: "If S is true, then God does not exist." We can abbreviate this as S = "If S then ¬G"

    Assume for a moment S is true. Then we have:
    S [assumed]
    If S then ¬G [meaning of S]
    ¬G [modus ponens]

    Assuming S implies ¬G. But this is the statement that "If S is true then God does not exist" which is S itself! Hence S *is* true, and the above argument applies. God does not exist.

    Still not convinced? Let's proceed by contradiction. Let's assume S is false.
    ¬S [Assumed]
    ¬(If S then ¬G) [meaning of S]
    ¬(¬S or ¬G) [material implication]
    S and G [De Morgan's rule]
    S [simplification]
    ¬S and S [conjunction]

    ...which is a contradiction. Incoherence and the end of reason.

    This is an easy example, because I made it so blatantly obvious what I was doing. But it illustrates the importance of tracing every step of the argument. It's not enough to say that S must be true or else we get a contradiction and the end of reason. You have to know how every step along the way works, and have tested its validity.

    ReplyDelete
  195. NiV,

    I think you are missing the context. What is happening is criticisms of naturalism are being made which show that it undermines reason, including the reason which must support it. Now, it is perfectly natural and sensible that, when faced with such a situation, we would suggest that naturalism must be wrong. That naturalism undermines reason, and therefore itself, is itself the reason for abandoning it.

    Our reasons for accepting reason are far stronger than for accepting a speculative metaphysics such as naturalism. The evidence for naturalism would have to be very close, or actually, self-evident before we would accept it over reason. This actually appears to be what some naturalists who at least feign preference for naturalism seem to imply - naturalism is just obviously true, basically self-evident, which is preposterous.

    There is no need to go into further speculation. A fatal flaw in naturalism has been pointed out, other directions than naturalism have been pointed towards.

    ReplyDelete
  196. I could add, only half-facetiously, that you'll be left with an even bigger flaw in your understanding if you do follow naturalism into incoherence. That is the point, all the talk of argument is rather ironic in this regard.

    The only way it would be anything but asinine to follow this line of speculation would be if naturalism had overwhelming evidence for it, so as to be near-self-evident, which is simply not the case.

    ReplyDelete
  197. Jeremy,

    Again, the problem is that Ross puts forward an argument against machine reasoning that apparently applies equally well to human reasoning. But he doesn't explain *how* the same argument fails to render human reasoning incoherent, he simply argues that it must do, because such a conclusion would be unacceptable.

    If the primary argument is valid, then it's *valid* - and human reasoning has to go too. If you can't accept that, then there must be something wrong with the argument as applied to humans, and if you don't know what that problem is, you can't be sure that it doesn't apply to the argument on machines as well.

    Ross acknowledges that the argument apparently applies to humans as well, which is why he addresses the point with the comments on human reason. But asserting that the conclusion in that case is obviously false doesn't get us out of the problem. It means there's still an unidentified problem or subtlety in the argument, which needs to be explained.

    ReplyDelete
  198. You seem to have confused the basic structure of Ross's argument. This would appear to be the primary problem at this point, I'm afraid.

    His argument is whether or not formal thinking is a physical process. He concludes it is not - because formal thinking must be determinate and physical processes are not - and, as a consequence, concludes that human reasoning is different to machine "reasoning".

    He supports the premise that formal thinking must be determinate, partly, by the fact that basic reason would reduced to incoherence by this. He takes this for granted for much the same reasons I have been talking about.

    It is simply incorrect to suggest Ross thinks the argument - the conclusion that physical processes do not engage in formal thinking - applies to humans: that is, I sorry, little short of a howler. A basic consequence of his argument is it doesn't apply to humans - humans do engage in formal thinking and are therefore more than just physical beings, or processes.

    After all the discussion, I'm struggling to see how you could be so confused on the fundamental structure and content of the argument.

    ReplyDelete
  199. Jeremy,

    The argument by which he concludes that physical processes are not determinate can be applied just as easily to humans. He doesn't say why it can't, he just rejects the conclusion.

    ReplyDelete
  200. He concludes it is not - because formal thinking must be determinate and physical processes are not - and, as a consequence, concludes that human reasoning is different to machine "reasoning".

    Is substance dualism the only logical alternative? In other words, does showing that metaphysical naturalism is incoherent* lead to a determinate logical solution about what replaces it or are you still left with contradictory meanings?

    *To repeat, my skepticism about the argument stems from the way in which other living animals utilize biological nonlinear processes to integrate various sensory inputs with learned and instinctive behaviors in a coordinated goal which rules out other functions. It is true that we do this at a much more abstract level, but I don’t deny humans are extraordinary machines, only that humans are supernatural.

    ReplyDelete